Anda di halaman 1dari 119

Anaesthesia & Radio

Anaesthesia & Radio

Marking: +1 for correct answer & -0 for incorrect.


Test Duration(mins) :

180

Time Left(mins) :

180

Instructions
Once the test has started, do not press the refresh button (or F5 on your keyboard)
Itisadvisable to save the test regularly to avoid losing yourinformation, save test refers
to storing the attempted part of the test.
In case of accidental failure of internet connectivity the system will save the attempted
portion of the test automatically.
Once the test time is over, you will be awardeda graceduration to wrap up the test and
"Submit" it.
Submittestrefers to the final completion of test; once you submit the testyou will not be
able to edit / preview youranswers.
Please ensure that you are connected to the internet, while submitting the test.

Section

Part
(Q.1) All are true about N2O except
Colour of cylinder is blue
(a)
Priestly was the first to prepare it
(b)
Stored in cylinder as gas
(c)
It is prepared by heating ammonium nitrate
(d)
Your Response :
Correct Answer
C
:
Stored in cylinder as gas
Exp:
It is stored as a liquid in the cylinder.
(Q.2) False about xenon anaesthesia?
Slow induction and slow recovery
(a)
Minimal cardiovascular side effects
(b)
Low solubility in blood
(c)

Explosive in nature
(d)
Your Response :
Correct Answer
A
:
Slow induction and slow recovery
Exp:
Ref. Morgans Anaesthesia 4th ed. 155

XENON- is an inert, Colourless and odourless gas with no irritation to the


respiratory tract. Well tolerated with gas induction
Low blood/gas(0.17) and oil/water partition co-efficients(20) allowing
rapid induction and recovery
Produces unconsciousness with analgesia and a degree of muscle
relaxation
MAC of 60-70% allows a reasonable inspired oxygen concentration
It does cause respiratory depression, to the point of apnoea.
It is cardiac stable. May attenuate the myocardial depressant effects of
isoflurane.
Not metabolised in the body and is eliminated rapidly and completely via
the lungs.
It is non toxic and is not associated with allergic reactions
Stable in storage, no interaction with anaesthesia circuits or soda lime.
Should not be used with rubber anaesthesia circuits as there is a high loss
through the rubber
Expensive - Routine usage will only be possible with a closed circuit
delivery system that recycles xenon.

Owing to environmental concerns there may be no alternative but to


use xenon even if it incurs an increase in cost.
(Q.3) All are actions of depolarising agents except:Bradycardia
(a)
Hypokalemia
(b)
Increased Intracranial tension
(c)
Increased Intraocular pressure
(d)
Your Response :
Correct Answer
B
:
Hypokalemia
Exp:
They causes hyperkalemia, and if hyperkalemia is already

present depolarizing agents are a contraindication.


(Q.4) Dose of atropine for premedication per kg body weight is:
10 mg
(a)
0.1 mg
(b)
0.01 mg
(c)
0.001 mg
(d)
Your Response :
Correct Answer
C
:
0.01 mg
Exp:
Doses of atropine:
For premedication: 0.01 mg/kg.
Along with neostigmine for preventing muscarinic side effects: 0.02 mg/kg.
Vagolytic dose: 2 mg (0.04 mg/kg).

(Q.5) Intubation induced laryngeal spasm can not be prevented by;


Local anaesthesia spray
(a)
Fentanyl
(b)
Diltiazem
(c)
Promethazine
(d)
Your Response :
Correct Answer
D
:
Exp:
Promethazine
Intubation induced reflex response which may manifests as
laryngospasm, bronchospasm, hypertension and cardiac
arrhythmias. To blunt this response following methods are utilized:
IV Xylocaine 2-3 minutes before intubation.
Xylocaine spray.
Increasing the depth of anaesthesia with inhalational agents.
Opioids (opioid of choice is sufentanil).
Beta blockers (esmolol).
Calcium channel blockers.
Promethazine can increase the chances of reflex disturbances.
(Q.6) True about fospropofol is all except:
(a)
Aqueous phosphorylated prodrug of propofol

(b)
More prolonged recovery profile than propofol
(c)
Pain at the injection site was not reported with fospropofol
(d)
Significantly more respiratory depression than propofol.
Your Response :
Correct Answer
D.
:
Exp:
Significantly more respiratory depression than propofol.
A water-soluble prodrug of propofol, fospropofol (Lusedra), has recently been
approved by the Food and Drug Administration (FDA) for monitored anesthesia
care sedation in adult patients undergoing diagnostic or therapeutic procedures.

Fospropofol is hydrolyzed to propofol, formaldehyde, and


phosphate. The formaldehyde and phosphate are comparable to
endogenous levels and may only be of concern in cases of overdose.
As fospropofol must first undergo metabolism to propofol, time to
onset is prolonged. Since fospropofol is preservative-free, each vial
is intended for single use and should be discarded after the bolus
injection is drawn up into a sterile syringe.
The elimination of propofol from a bolus administration of
fospropofol is also slower than after bolus administration of
propofol in lipid emulsion.
This slower elimination might arguably be advantageous in
requiring only a single dose to provide anesthesia for short
procedures. However, a slower onset of fospropofol when compared
to lipid formulated propofol could also lead novice users to redose
fospropofol before the peak effect has occurred.
This could lead to excessive second peaks and also cause delayed
emergence at the end of short procedures.
Pharmacodynamic studies show fospropofol provides clinically
useful sedation and EEG/BIS suppression while causing
significantly less respiratory depression than propofol.
Pain at the injection site, a common problem with propofol is not
reported with fospropofol; the major patient complaint with its use is
transitory perianal itching.
(Q.7) Urgent reversal of warfarin therapy can be done by administration of:
Cryoprecipitate
(a)
Platelets
(b)
Fresh frozen plasma
(c)
Packed red blood cells
(d)
Your Response :
Correct Answer C

:
Exp:

Fresh frozen plasma


Oral anticoagulants should be stopped 7 days prior to elective
surgery. But during emergency rapid reversal can be done with
vitamin K a day before surgery or with fresh frozen plasma on the
day of surgery.

(Q.8) ETHER DAY is celebrated on?


October 20
(a)
October 16
(b)
November 20
(c)
November 16.
(d)
Your Response :
Correct Answer
B
:
October 16.
Exp:
Ether Day
William Morton (18191868) was a dentist and partner of Horace Wells's. After
a taking a course in anesthesia from Wells, Morton left the partnership in Hartford
and established himself in Boston.
He continued his interest in anesthesia, but with diethyl ether replacing nitrous
oxide. Ether proved a good choice as it supports respiration and the cardiovascular
system at analgesic levels, and is potent enough to administer in room air without
hypoxia.
He practiced the administration of ether on a dog and then used it when extracting
teeth from patients in his office.
On October 16, 1846, Morton gave the first public demonstration of ether as an
anesthetic for Johns Collins Warren, distinguished surgeon and a founder of the
Massachusetts General Hospital.
In attendance in the surgical amphitheater were several surgeons, medical
students, and a newspaper reporter.
After anesthesia was induced using a makeshift inhaler, Warren successfully
removed a vascular mass from the patient's neck with no ill effects.
Warren was an originator of the Boston Medical and Surgical Journal (now The
New England Journal of Medicine), and by November 1846 the demonstration
was published in an article by Henry J. Bigelow. 3 The stature of Warren and
Bigelow lent considerable credence to the advent of surgical anesthesia; as news
spread rapidly, surgeons around the world were quick to adopt this "American
invention."
The Massachusetts General Hospital has restored and preserved the original
amphitheater where the demonstration took place, now called the Ether Dome.
It is designated as a Registered National Historic Landmark commemorating the
first public demonstration, rather than discovery, of the use of ether as an
anesthetic.
The day of OCTOBER 16th is now celebrated as the ETHER DAY in the
memory of this achievement.

(Q.9) Brain temperature is most accurately measured through:


(a)
Tympanic membrane
(b)
Nasopharynx
(c)
Scalp
(d)
Cornea
Your Response :

Correct Answer
A
:
Tympanic Membrane
Exp:
Nasopharynx and tympanic membrane both gives accurate measurement of brain
temperature due to its close proximity to hypothalamus. Although tympanic membrane
is more accurate than nasopharynx but the presence of wax alters the value and
increased chances of perforation of tympanic membrane nasopharynx is more
frequently used.

(Q.10) Latest mode of ventilation in patient with poor oxygenation in ARDS


PCV
(a)
Reverse ratio ventilation
(b)
Liquid ventilation
(c)
Prone ventilation
(d)
Your Response :
Correct Answer
C
:
Liquid ventilation
Exp:
Liquid ventilation (LV) is a technique of mechanical ventilation in which the
lungs are insufflated with an oxygenated perfluorochemical liquid rather than an
oxygen-containing gas mixture. The use of perfluorochemicals, rather than
nitrogen, as the inert carrier of oxygen and carbon dioxide offers a number of
theoretical advantages for the treatment of acute lung injury, including:

Reducing surface tension by maintaining a fluid interface with


alveoli
Opening of collapsed alveoli by hydraulic pressure with a lower
risk of barotrauma
Providing a reservoir in which oxygen and carbon dioxide can be
exchanged with pulmonary capillary blood
Functioning as a high efficiency heat exchanger
(Q.11) A 10oC decrease in temperature causes decrease in cerebral metabolic rate by:
(a)
10%
(b)
30%
(c)
50%
(d)
70%
Your Response :
Correct Answer
D
:
Exp:
70%
With each degree fall in temperature the metabolic rate is
reduced by 7%, so by reducing temperature by 10C the metabolic
rate can be reduced by 70% and cerebral cortex can be protected
for 10 minutes at 30C and for 1 hour at 15C.

(Q.12) Optimal operation theatre temperature should be:


(a)
21C
(b)
28C
(c)
32C
(d)
37C
Your Response :
Correct Answer
A
:
21C
Exp:
Optimal operation theatre temperature for adults is 21C and for children is 28C.

(Q.13) Identify the object

Guedels oropharyngeal airway


(a)
Disposable Nasopharyngeal airway
(b)
Laryngeal mask airway
(c)
Tracheal tube
(d)
Your Response :
Correct Answer
A
:
Guedels oropharyngeal airway
Exp:

(Q.14) In a match test the distance between lighted paper and mouth should be:
10 cm
(a)
15 cm
(b)
200 cm
(c)
25cm
(d)
Your Response :
Correct Answer
B
:
Exp:
15 cm
A person with normal pulmonary function should be able to blow
off a match stick or lighted paper from a distance of 15 cm.

(Q.15) Fastest route of absorption of local anaesthetic is?


(a)
Intercostal
(b)
Epidural
(c)
Brachial
(d)
Caudal
Your Response :
Correct Answer
A
:
Exp:
Intercostal
Ref-text book of anesthesia by MILLER -chapter; regional
block
The fastest route for absorption of LA is intercostal block, due to
close location of blood vessel around the nerve, so that is why LA
are rapidly taken by in intercostal block.
(Q.16) Local anaesthetic agent which has antimuscarinic action on heart muscle receptors is:
Procaine
(a)
Cocaine
(b)
Chloroprocaine
(c)
None of the above
(d)
Your Response :
Correct Answer
B
:
Exp:
Cocaine
Cocaine is a sympathomimetic (and thus antimuscarinic) local
anaesthetic and thus adrenaline should not be added to it.
(Q.17) The advantage of using epidural opioids over local anaesthetics are all except:
(a)
Less hypotension
(b)
No motor block
(c)
Frequent injections not required
(d)
Less expensive
Your Response :
Correct Answer
D
:
Exp:
Less expensive
The main drawback of local anaesthetics is that there is possibility of motor
blockade if higher concentration is used and motor paralysis is not acceptable in
postoperative period. Other advantages are no sympathetic block so hypotension
is less and after one single dose of morphine analgesia may last for 12-16 hours.

(Q.18) About lidocaine all are true, except:


Dose need to altered in renal failure
(a)
Loading dose before infusion
(b)
It is little effected by heat & pH
(c)

Prolongs refractory period


(d)
Your Response :
Correct Answer
A
:
Exp:
Dose need to altered in renal failure

Local anesthetics are poorly water soluble and this limits their renal
excretion (of unchanged drug) to less than 5%. For treating
ventricular arrhythmias initial loading dose is to be followed by
infusion. The amides are so stable that they are not effected even
by autoclaving. Effective refractory period (of cardiac tissue) is
prolonged by local anaesthetics.
(Q.19) Which of the following local anesthetic is more safe in surface and infiltrating anesthesia:
Procaine
(a)
Cocaine
(b)
Lignocaine
(c)
Amethocaine
(d)
Your Response :
Correct Answer
C
:
Lignocaine
Exp:
Lignocaine and prilocaine are considered as safe local anaesthetic and can be used
for both topical and infiltration (local nerve blocks) anaesthesia.

(Q.20) A moribund patient who is not expected to survive without an operation is put under which
category as per the ASA Physical status classification system ?

(a)
(b)
(c)
(d)
Your Response :
Correct Answer
C
:
Exp:
P5.

P2
P4
P5
P6

Table : ASA Physical Status Classification System


A normal healthy patient
A patient with mild systemic disease
A patient with severe systemic disease
A patient with severe systemic disease that is a constant threat
to life
P5 A moribund patient who is not expected to survive without
the operation
P6 A declared brain-dead patient whose organs are being
removed for donor purposes
P1
P2
P3
P4

Criticism of the ASA scale is primarily due to its exclusion of age


and difficulty of intubation (discussed later in this chapter). Cullen
and associates examined 1095 patients undergoing total hip
replacement, prostatectomy, or cholecystectomy, and found that
both age and ASA scale accurately predicts postoperative
morbidity and mortality 17
(Table 46-7). The ASA scale remains useful and should be applied
to all patients during the preoperative visit.
Table : ASA Physical Status and
Mortality
Score
P1
P2
P3
P4
P5

Mortality (%)
0.1
0.2
1.8
7.8
9.4

(Q.21) Significant extrahepatic metabolism is shown by:


(a)
Prilocaine
(b)
Lignocaine
(c)
Bupivacaine
(d)
Ropivacaine
Your Response :
Correct Answer
A
:
Exp:
Prilocaine
Prilocaine other than hepatic metabolism is also metabolized
significantly in lungs.
Biologic Properties of Commonly Used Local Anesthetics
Agent

Equianesthetic
Concentration

Esters
Procaine
2
Chloroprocaine 2
Tetracaine
0.25
Amides
Prilocaine
1
Lidocaine
1
Mepivacaine 1
Bupivacaine 0.25

Approximate
Anesthetic
Duration (min)

Site of
Metabolism

50
45
175

Plasma
Plasma
Plasma

100
100
100
175

Liver/lung
Liver
Liver
Liver

Ropivacaine
Etidocaine

0.3
0.25

150
200

Liver
Liver

(Q.22) Local anaesthetic causing malignant hyperthermia is:


(a)
Lignocaine
(b)
Prilocaine
(c)
Bupivacaine
(d)
Dibucaine
Your Response :
Correct Answer
A
:
Exp:
Lignocaine
Lignocaine by releasing calcium from sarcoplasmic retinaculum
can cause malignant hyperthermia in susceptible individuals.
(Q.23) Concentration of adrenaline used with local anaesthetic:
(a)
1:1,000
(b)
1:10,000
(c)
1:1,00000
(d)
1:2,00000
Your Response :
Correct Answer
D
:
Exp:
1:2,00000
Concentration of adrenaline used for local anaesthetic is 1 in 2
lakhs, (that of phenylephrine is 1 in 20,000).
(Q.24) Maximum safe dose of bupivacaine:
(a)
1 mg/kg
(b)
2 mg/kg
(c)
3 mg/kg
(d)
5 mg/kg
Your Response :
Correct Answer
B
:
Exp:
2 mg/kg
Maximum safe dose of bupivacaine is 2 mg/kg with or without
adrenaline.
(Q.25) Drug of choice for ventricular arrhythmias induced by bupivacaine:
(a)
Bretylium tosylate
(b)
Lignocaine
(c)
Nifedipine
(d)
Verapamil

Your Response :
Correct Answer
A
:
Exp:
Bretylium tosylate
Bupivacaine has got high cardiotoxic potential. Treatment for
ventricular arrhythmia produced by bupivacaine is bretylium.
(Q.26) False about ropivacaine:
(a)
Less cardiotoxic than bupivacaine
(b)
More cardiotoxic than lignocaine
(c)
Contains only one (S) enantiomer
(d)
Onset of action is faster then bupivacaine
Your Response :
Correct Answer
D
:
Onset of action is faster then bupivacaine
Exp:
Cardiotoxicity of bupivacaine is because of R isomer (bupivacaine
is racemic mixture of S and R isomers).
Ropivacaine contains only S enantiomer so cardiotoxicity is less
than bupivacaine (but still higher than lignocaine).
Anaesthetic properties (onset, intensity; duration of I action) of
ropivacaine are almost similar to bupivacaine.
(Q.27) Intravenous anaesthetic metabolized by pseudo cholinesterase:
(a)
Propanidid
(b)
Althesin
(c)
Eltanolone
(d)
Clonidine
Your Response :
Correct Answer
A
:
Exp:
Propanidid
Propanidid is no more used in anaesthesia due to severe allergic
reactions. It is metabolized by pseudocholinesterase.
(Q.28) In doing phrenic nerve block, it is best to infiltrate:
Scalenus anterior
(a)
Scalenus posterior
(b)
Posterior border of sternomastoid
(c)
Anterior border of sternomastoid
(d)
Your Response :
Correct Answer
C
:

Exp:

Posterior border of sternomastoid


Phrenic nerve is blocked 3 cm above the clavicle at the posterior
border of sternomastoid.

(Q.29) In a patient with thyroid surgery, the block to be given is:


Stellate ganglion
(a)
Upper cervical ganglion
(b)
Dorsal sympathetic ganglion
(c)
Any of the above
(d)
Your Response :
Correct Answer
B
:
Upper cervical ganglion
Exp:
Cervical plexus block is most commonly given for carotid endarterectomy but
thyroidectomy and tracheostomy can be done under bilateral cervical plexus
block.

(Q.30) Shortest acting synthetic opioid is


Fentanyl
(a)
Remifentanil
(b)
Alfentanil
(c)
Sufentanil
(d)
Your Response :
Correct Answer
B
:
Remifentanil
Exp:
Remifentanilis a potent ultra short-acting syntheticopioidanalgesicdrug.
It is given to patients during surgery to relieve pain and as an adjunct to
ananaesthetic.
Remifentanil is used forsedationas well as combined with other medications for
use ingeneral anesthesia.
The use of remifentanil has made possible the use of high dose opioid and low
dose hypnotic anesthesia, due to synergism between remifentanil and various
hypnotic drugs and volatile anesthetics.

(Q.31) Interscalene approach to brachial plexus block does not provide optimal surgical anaesthesia in
the area of distribution of which of the following nerve:

Ulnar
(a)
Median
(b)
Radial
(c)
Musculocutaneous
(d)
Your Response :
Correct Answer
A
:

Exp:

Ulnar
Ulnar nerve is usually spared when brachial plexus is blocked
through interscalene approach.

(Q.32) Highest incidence of pneumothorax occurs when branchial plexus is blocked by


which route:
(a)
Interscalene
(b)
Supraclavicular
(c)
Infraclavicular
(d)
Axillary
Your Response :
Correct Answer
B
:
Exp:
Supraclavicular
The incidence of pneumothorax may be as high as to 6% when
brachial plexus is blocked through supraclavicular route but
fortunately most of these pneumothorax resolve by themselves
without treatment.
(Q.33) The following combination of drugs is best suited for day care surgery :
(a)
Atracurium, morphine, isoflurane
(b)
Mivacurium, fentanyl, sevoflurane
(c)
Atracurium, remifentanil, halothane
(d)
Mivacurium, pethidine, desflurane
Your Response :
Correct Answer
B
:
Exp:
Mivacurium, fentanyl, sevoflurane
Ideal agent for day care procedures includes those which are
eliminated sooner from the body and provides a clear headed
recovery.
The best combination in this regard is the propofol,
benzodiazepines like midazolam and opioids like Fentanyl,
alfentanil or remifentanil. Among inhalational agents best suited is
desflurane, although sevoflurane is equally advantageous.
Mivacurium: shortest acting non depolarizing muscle relaxant
metabolized by pseudocholinesterase. It does not cross blood
brain barrier and placenta so can be safely used. S/E include
hypotension, flushing and rarely bronchospasm. But best is
gantacurium (if available in choices)
Sevoflurane: B/G coefficient 0.69 so fastest, pleasant and smooth
induction with no significant systemic toxicity. Best is xenon and

then desflurana if in choices


(Q.34) For brachial plexus block needle is inserted:
(a)
Medial to subclavian artery
(b)
Lateral to subclavian artery
(c)
Medial to subclavian vein
(d)
Lateral to subclavian vein
Your Response :
Correct Answer
B
:
Exp:
Lateral to subclavian artery
The needle should be lateral to avoid pneumothorax as dome of
pleura is on medial side. The point of injection is the midpoint of
clavicle which is taken as point at which subclavian artery is felt or
a line extended straight onward to external jugular vein.
(Q.35) In ankle block all of the nerves are blocked, except:
(a)
Superficial peroneal
(b)
Deep peroneal
(c)
Saphenous
(d)
Common peroneal
Your Response :
Correct Answer
D
:
Exp:
Common peroneal
In ankle block deep peroneal, superficial peroneal and saphenous
nerve are blocked at ankle.
(Q.36) The nerve root blocked in pudendal nerve block:
(a)
L2-3
(b)
L2-4
(c)
S2-4
(d)
L5, S1
Your Response :
Correct Answer
C
:
Exp:
S2-4
Pudendal nerve block, blocks sacral segments (824) thus provides
perineal analgesia.
(Q.37) The effect of chilling in refrigeration analgesia includes:
Interference with conduction of nerve impulse
(a)
Reduction of metabolic rate and oxygen requirement
(b)
Inhibition of bacterial growth and infection
(c)
All of the above
(d)

Your Response :
Correct Answer
D
:
Exp:
All of the above
For chilling analgesia (cryoanalgesia) CO2 or N2O is used. At such
low temperatures there is inhibition of nerve conduction producing
analgesia.
Hypothermia decreases metabolic rate and oxygen consumption of
tissue.
Hypothermia inhibits the bacterial growth. Healing is also retarded
at low temperatures.
(Q.38) Which of the following drug can be accumulated in foetus in very significant
amount if given to pregnant mother:
(a)
Thiopentone
(b)
Propofol
(c)
Midazolam
(d)
Lignocaine
Your Response :
Correct Answer
D
:
Exp:
Lignocaine
Local anaesthetic and opioids are bases. Once they reach the fetus
they become non-ionized (due to highly acidic pH) and can not
come back to maternal circulation leading to toxic accumulations in
fetus.
(Q.39) Most preferred technique for painless labour:
Lumbar epidural
(a)
Sacral epidural
(b)
Lumbar + sacral epidural (double catheter technique)
(c)
(d)
Para cervical block
Your Response :
Correct Answer
A
:
Exp:
Lumbar epidural
Nowadays for painless labour continuous lumbar epidural (i.e.,
through epidural catheter) is employed. Double catheter
technique is now obsolete because when in both stages
analgesia is possible through one catheter why to put another
catheterr.
Secondly chances of contamination of sacral catheter is very high.

Paracervical block can produce analgesia for only 1 st stage while


pudendal nerve block provides analgesia only for second stage.
Nitrous oxide inhalation may produce fetal diffusion hypoxia and
analgesia is not satisfactory.
(Q.40) Among patients who require nutritional resuscitation in an intensive care unit,
the best evidence that nutritional support is adequate is
(a)
Urinary nitrogen excretion levels
(b)
Total serum protein level
(c)
Serum albumin level
(d)
Serum transferrin levels
Your Response :
Correct Answer
C
:
Exp:
Ref: (Schwartz, 7/e, pp 3646.)
The serum albumin level provides a rough estimate of protein
nutritional adequacy.
The accuracy of this estimate is affected by the long half-life of
albumin (3 wk) and vagaries of hemodilution.
The acute-phase serum proteins have a very short half-life (hours)
and may also provide good short-term indications of nutritional
status.
Transferrin is one of these acute-phase proteins, but unfortunately
its levels too are influenced by changes in intravascular volume
and, along with the other acute-phase reactants, rise nonspecifically
during acute illness.
All the listed responses provide some useful information about
nutrition and adequacy of replacement.
(Q.41) Last to recover in spinal anaesthesia:
Pain
(a)
Motor
(b)
Proprioception
(c)
Preganglionic sympathetic
(d)
Your Response :
Correct Answer
D
:
Exp:
Preganglionic sympathetic
Order of paralysis is autonomic sensory motor and the recovery
is in reverse orders i.e., motor sensory autonomic but this is
controversial and many studies have suggested that autonomic
activity returns before sensory.

(Q.42) Best way to prevent hypotension during spinal anaesthesia:


(a)
Preloading with crystalloids
(b)
Dopamine
(c)
Mephentermine
(d)
Trendelenburg position
Your Response :
Correct Answer
A
:
Preloading With Crystalloids
Exp:
Best method to prevent hypotension is to preload the patient with crystalloids or
colloids. B, C, D are used as a part of treatment not preventive measures.

(Q.43) Post spinal headache can be prevented by:


Use of thinner needle
(a)
NSAIDs
(b)
Preanesthetic medication
(c)
Plenty of oral fluids
(d)
Your Response :
Correct Answer
A
:
Exp:
Use of thinner needle
The most important causative factor for spinal headache is needle
size, so use of small gauge needle is the, AJ best method to prevent
post spinal headache.
(Q.44) Most common causative agent for meningitis after spinal subarachnoid block is:
Staphylococcus epidermidis
(a)
Staphylococcus aureus
(b)
Pseudomonas
(c)
Streptococcus
(d)
Your Response :
Correct Answer
A
:
Exp:
Staphylococcus epidermidis
Infective meningitis is mainly due to Staphylococcus epidermidis
carried along with needle from skin.
(Q.45) All of the following drugs can be given in spinal, except:
(a)
Morphine
(b)
Ketamine
(c)
Bupivacaine
(d)
Etomidate
Your Response :
Correct Answer
D
:

Exp:

Etomidate
Drug which can be given through spinal are local anaesthetics,
opioids and preservative free ketamine.

(Q.46) Tuohy's needle is used for:


(a)
Spinal block
(b)
Epidural block
(c)
Saddle block
(d)
Brachial plexus block
Your Response :
Correct Answer
B
:
Exp:
Epidural block
Tuohy's needle used for epidural anaesthesia has blunt bevel with
15 to 30 curve at tip.
(Q.47) Blood should not be mixed with:
(a)
Ringer lactate
(b)
Normal saline
(c)
Hypertonic saline
(d)
All of the above
Your Response :
Correct Answer
A
:
Exp:
Ringer lactate
Haemaccel and Ringer lactate contains calcium so should not be
mixed with blood as it contains citrate.
(Q.48) First to be blocked by muscle relaxants:
(a)
Laryngeal muscles
(b)
Diaphragm
(c)
Thenar muscles
(d)
Intercostals
Your Response :
Correct Answer
A
:
Exp:
Laryngeal muscles
First muscles to be blocked by muscle relaxants (both depolarizing
and non depolarizing type) are central muscles i.e., muscles of head
and neck (face, jaw, pharynx, larynx), respiratory, abdominal
muscles and muscles of trunk. After the central muscles, muscles of
limbs (peripheral muscles) are blocked.
(Q.49) First to recover after muscle relaxants:

(a)
Laryngeal muscles
(b)
Abdominal muscles
(c)
Diaphragm
(d)
Thenar muscles
Your Response :
Correct Answer
C
:
Exp:
Diaphragm
The sequence of recovery is in the same way as it goes i.e., first to
recover are central muscles like larynx and diaphragm and then
limb muscles.
(Q.50) Most useful supraglottic airway device in abdominal surgery
FAST-trach LMA
(a)
PROSEAL LMA
(b)
SLIPA
(c)
C TRACH LMA
(d)
Your Response :
Correct Answer
B
:
PROSEAL LMA
Exp:
We understand that clinicians sometimes select an ET tube for optimum airway
protection when LMA patient-friendly characteristics would be desirable. To
enhance supraglottic airway protection and extend LMA benefits to a greater
number of patients we developed the ProSeal LMA. The added features of the
ProSeal LMA include:

A softer silicone cuff reducing the likelihood of throat irritation and


stimulation
High seal pressure - up to 30 cm H20 - Providing a tighter seal
against the glottic opening with no increase in mucosal pressure
Provides more airway security
Enables use of PPV in those cases where it may be required transient or extended, planned or unplanned
A built-in drain tube designed to channel fluid away and permit
gastric access for patients with GERD or during extended cases
where endotracheal intubation is not required
Ability to realize the benefits of spontaneous ventilation more often
Optional Insertion tool
(Q.51) Pseudocholinesterase is synthesized by:
(a)
Liver

(b)
Plasma
(c)
Muscle end plate
(d)
Nerve ending
Your Response :
Correct Answer
A
:
Exp:
Liver
Pseudocholinesterase which metabolizes succinylcholine is
synthesized by liver and is present in plasma. True cholinesterase
which metabolizes acetylcholine is synthesized by muscle end
plate.
(Q.52) Not a contraindication of succinylcholine:
(a)
Recent burns
(b)
Recent cerebral stroke
(c)
Recent crush injury
(d)
Recent myocardial ischemia
Your Response :
Correct Answer
D
:
Exp:
Recent myocardial ischemia
A injured and regenerating nerve develops extra junctional
receptors which are very sensitive to depolarizing muscle relaxant
(suxamethonium) and resistant to nondepolarizers. Due to this
reason succinylcholine is contraindicated in:
Burns up to 1 year.
Hemiplegia/paraplegia up to 6 months. J. Crush injury up to 3
months.
(Q.53) Phase I blocker is:
Suxamethonium
(a)
Atracurium
(b)
Gallamine
(c)
D-Tubocurare
(d)
Your Response :
Correct Answer
A
:
Exp:
Suxamethonium
The kind of depolarizing block produced by succinyl choline is
phase I block.
(Q.54) Which of the following is not true of non depolarizing skeletal muscle relaxants:
Causes histamine release
(a)

Hypothermia prolongs their effect.


(b)
Fasciculations of muscle
(c)
Ganglion blockade.
(d)
Your Response :
Correct Answer
C
:
Exp:
Fasciculations of muscle

Fasciculations are shown by depolarizing muscle relaxants. These


agents do not cause hypothermia but their effect may be prolonged
in hypothermia.
(Q.55) Histamine release is one of the common features with administration of:
Atracurium
(a)
Adrenaline
(b)
Thiopentone
(c)
Ketamine
(d)
Your Response :
Correct Answer
A
:
Exp:
Atracurium
Non depolarizers are classified as steroidal (pancuronium,
vecuronium, rocuronium, rapacuronium, pipercuronium) and
benzylisoquinolones (d- TC, metocurine, doxacurium, atracurium,
cis-atracurium, mivacurium). Benzylisoquinolones release
histamine.
(Q.56) Untrue about mivacurium:
(a)
Metabolized by pseudocholinesterase
(b)
Releases histamine
(c)
Onset of action is early
(d)
Short duration of action
Your Response :
Correct Answer
C
:
Onset of action is early
Exp:
Mivacurium is the non-depolarizing muscle relax, metabolized by
pseudocholinesterase. The main advantage is its short duration of action (5-10
minutes) but onset is slow (2-3 minutes).

(Q.57) Train of four is characteristically used in concern with


(a)
Malignant hyperthermia
(b)
Non-depolarizing neuromuscular blockers
(c)
Mechanical ventilation
(d)
To check hemodynamic parameters
Your Response :

Correct Answer
B
:
Exp:
Non-depolarizing neuromuscular blockers
The degree of neuromuscular blockade is monitored by applying
various patterns of electrical stimulation. A twitch is a single pulse
that is delivered from every 10 s (1-0.1 Hz). Increasing block
results in decreased evoked response to stimulation.
Train-of-four stimulation devotes four successive 200 s stimuli
in 2 s (2 Hz). The twitches in a train-of-four pattern progressively
fade as relaxation increases. The ratio of the responses to the first
and fourth twitches is a sensitive indicator of nondepolarizing
muscle paralysis. Disappearance of the fourth twitch represents a
75% block, the third twitch an 80% block, and the second twitch a
90% block. Clinical relaxation usually requires 75-95%
neuromuscular blockade.
(Q.58) All of the following may be used for premedication except:
(a)
Diazepam
(b)
Pethidine
(c)
Atropine
(d)
Pancuronium
Your Response :
Correct Answer
D
:
Exp:
Pancuronium
Diazepam (benzodiazepine) is used for relieving anxiety, inducing
good sleep and producing amnesia. Opioids for producing
analgesia and atropine (anticholinergics) for drying up the
secretions. Pancuronium is non-depolarizing muscle relaxant
used during surgery to produce muscle relaxation.
(Q.59) All of the following features distinguish infant larynx from adult EXCEPT:
Epiglottis is long and leafy
(a)
Subglottic region is narrowest laryngeal portion
(b)
Large tongue
(c)
Lower placement of the larynx
(d)
Your Response :
Correct Answer
D
:
Exp:
Lower placement of the larynx
Following features of infant larynx in contrast to adult larynx:
Epiglottis is long and leafy.

Subglottic region is the narrowest portion of larynx.


Larynx is placed at a higher level (in adults, it is placed at the level
of C3-C6 vertebrae).
Anatomic Differences between the Child's and Adult's
Larynx
Child's larynx
Adult's larynx
Size
Smaller
Larger
Shape
Lumen is funnel-shaped with the Narrowest part of
narrowest part below the vocal lumen is at the
cords and within the cricoid ring vocal cords
Location Higher, closer to the tongue base; Vertical extent is
vertical extent is opposite C3, C4, lower, opposite C4,
C5 vertebrae; more anterior
C5, C6 vertebrae
Epiglottis Longer, narrower, and "U"
Shorter and wider
shaped; the angle between glottis
and epiglottis is more acute;
increased chance of airway
obstruction (see Figure 1-8)
Vocal
Angled in relation to the axis of Perpendicular to the
cords
trachea; shorter; more
axis of trachea
cartilaginous; more distensible;
more likely to be injured
Rigidity The laryngeal cartilages are softer More rigid
and more pliable
Response Mucous membrane is more
Less vulnerable to
to trauma loosely attached and swells more trauma and
readily when traumatized or
infection
infected
(Q.60) Most cardiotoxic local anaesthetic agent
Bupivacaine
(a)
Ropivacaine
(b)
Dibucaine
(c)
Levobupivacaine
(d)
Your Response :
Correct Answer
A
:
Bupivacaine
Exp:
DOC for hyperbaric spinal Anaesthesia as it is most potent
C/I in intravenous regional Anaesthesia, as it is most cardiotoxic
(VT, cardiac depression)
Very popular in obstetrics.

Adequate analgesic without significant motor block, causes


active co-operation of mother in vaginal delivery.
High lipid solubility causes more conc. Of drug in tissue then in
blood so less likely to reach fetus and produce neonatal
depression. (Due to low materno fetal transfer)
Not used for surface Anesthesia.
Bupivacaine is more prone to prolong QTc interval and induce
ventricular tachycardia or cardiac depression should not be used for
intravenous regional analgesia (Biers).
Levobupivacaine is equally potent but less cardiotoxic and less
prone to cause seizures then bupivacaine.
(Q.61) Total Cerebral Metabolic failure occurs at cerebral blood flow of
(a)
10 mL/100mg/min
(b)
20 mL/100mg/min
(c)
30 mL/100mg/min
(d)
40 mL/100mg/min
Your Response :
Correct Answer
A.
:
Exp:
10 ml/100mg/min
Ref. Morgan Anesthesia- Page 615.
Cerebral blood flow varies with metabolic activity. It is most
commonly measured with a gamma emitting isotope such as xenon.
CBF parallel with metabolic activity can vary from 10 -300
mL/100mg/min. average is 50 mL/100mg/min, grey matter it is 80
mL/100mg/min white matter it is 20 mL/100mg/min. total CBF
average 750 ml/min(15to 20%).
20-25 mL/100mg/min- cerebral impairment starts, EEG slows
15- 20 mL/100mg/min- flat EEG
10 mL/100mg/min- IRREVERSIBLE DAMAGE
(Q.62) Succinylcholine is short acting due to:
(a)
Rapid excretion
(b)
Poor absorption
(c)
Rapid hydrolysis
(d)
None of the above
Your Response :
Correct Answer C

:
Exp:

Rapid hydrolysis
Succinylcholine is rapidly metabolized in plasma by
pseudocholinesterase.

(Q.63) Blood gas partition coefficient of anaesthetic agent tells about:


Solubility in blood
(a)
Potency of agent
(b)
Time lag of induction of anaesthesia
(c)
All of the above
(d)
Your Response :
Correct Answer
C
:
Time lag of induction of anaesthesia
Exp:
Blood gas solubility (blood gas partition coefficient) is the best indicator for
induction and recovery. Agent with less blood gas solubility will have fast induction
and recovery while agents with less blood gas solubility will have slow induction

(Q.64) Side effect of halothane countered by atropine is:


Bradycardia
(a)
Hypotension
(b)
Hyperthermia
(c)
Constipation
(d)
Your Response
:
Correct Answer
A
:
Bradycardia
Exp:
Halothane usually causes sinus or nodal bradycardia which is reversed by atropine.

(Q.65) Test normally undertaken to demonstrate the presence of collateral circulation of hand is
Kety Schmidt test
(a)
Allen's test
(b)
Sellicks test
(c)
Swan Ganz test
(d)
Your Response :
Correct Answer
B
:
Allen's test
Exp:
Allen's test is done to determine the presence of collateral circulation of hand before
cannulating radial artery.

(Q.66) Full soaked sponge indicates blood loss of:


100 ml
(a)
200 ml
(b)
300 ml
(c)

(d)
Your Response :
Correct Answer
A
:
100 ml
Exp:

500 ml

A fully soaked sponge indicates a loss of 100-120 ml of blood, a fully soaked swab
indicates 20-25 ml while a fist of clots means 200-300 ml of loss.

(Q.67) Not a contraindication for halothane:


Head injury
(a)
Aortic stenosis
(b)
Previous history of hepatitis A infection
(c)
Pheochromocytoma
(d)
Your Response :
Correct Answer
C
:
Exp:
Previous history of hepatitis A infection
CI for halothane are:
Previous history of halothane induced hepatitis.
Head injury and intracranial lesions (as it raises ICT significantly).
Pheochromocytoma as it sensitizes myocardium to adrenaline.
Fixed cardiac output lesions like AS and MS.
Preexisting liver disease is not considered as CI as the main cause
of halothane hepatitis is outside the liver (immunologic basis).
(Q.68) All of the following factors decrease the CO2 absorption though the soda lime, except:
Increased flow
(a)
Increased dead space
(b)
Increased tidal volume
(c)
Increased resistance in circuit
(d)
Your Response :
Correct Answer
D
:
Increased resistance in circuit
Exp:
High flow allows less time for absorption. Larger tidal volume will pass without
carbon dioxide being absorbed. Increased dead space will decrease CO2 absorption.
Other factors which decrease the CO2 absorption are exhausted sodalime, loosely
filled soda lime or too tightly filled sodalime (this will decrease air space). Increasing
resistance to outflow will permit the gases to remain in contact of sodalime for more
time allowing more absorption.

(Q.6 Hypothermia may be defined as core body temperature less than:


38C
(a)

9)

37oC
(b)
36C
(c)
35oC
(d)
Your Response :
Correct Answer
D
:
Ex 35oC
p: Accidental hypothermia occurs when there is an unintentional drop in the body's core temperature

below 35C (95F). At this temperature, many of the compensatory physiologic mechanisms to
conserve heat begin to fail. Primary accidental hypothermia is a result of the direct exposure of a
previously healthy individual to the cold. The mortality rate is much higher for those patients who
develop secondary hypothermia as a complication of a serious systemic disorder.

According to HARRISON 17th Ed chapter 20 :


Physiologic Changes Associated with Accidental Hypothermia

Central
Severi Body
Temperat Nervous
ty
ure
System
Mild 35C
Linear
(95F) depression
32.2C of cerebral
(90F) metabolism
; amnesia;
apathy;
dysarthria;
impaired
judgment;
maladaptiv
e behavior

Moder <32.2C
ate
(90F)
28C
(82.4F)

Severe <28C
(82.4F)

Cardiovascu Respiratory Renal and


lar
Endocrine

Neuromusc
ular

Tachycardia, Tachypnea, Diuresis;


Increased
then
then
increase in preshiverin
progressive progressive catecholami g muscle
bradycardia; decrease in nes, adrenal tone, then
cardiacrespiratory steroids,
fatiguing
cycle
minute
triiodothyro
prolongation volume;
nine and
;
declining
thyroxine;
vasoconstric oxygen
increase in
tion;
consumption metabolism
increase in ;
with
cardiac
bronchorrhe shivering
output and a;
blood
bronchospas
pressure
m
EEG
Progressive Hypoventila 50%
Hyporeflex
abnormaliti decrease in tion; 50% increase in ia;
es;
pulse and
decrease in renal blood diminishin
progressive cardiac
carbon
flow; renal g
depression output;
dioxide
autoregulati shiveringof level of increased
production on intact; induced
consciousn atrial and
per 8C drop impaired
thermogene
ess;
ventricular in
insulin
sis; rigidity
pupillary arrhythmias; temperature; action
dilatation; suggestive absence of
paradoxical (J- wave)
protective
undressing; ECG
airway
hallucinatio changes
reflexes
ns
Loss of
Progressive Pulmonic Decrease in No motion;
cerebrovasc decrease in congestion renal blood decreased

ular
blood
autoregulati pressure,
on; decline heart rate,
in cerebral and cardiac
blood flow; output; recoma; loss entrant
of ocular dysrhythmia
reflexes;
s; maximum
progressive risk of
decrease in ventricular
EEG
fibrillation;
asystole

and edema; flow


75%
parallels
decrease in decrease in
oxygen
cardiac
consumption output;
; apnea
extreme
oliguria;
poikilother
mia; 80%
decrease in
basal
metabolism

nerveconduction
velocity;
peripheral
areflexia;
no corneal
or
oculocepha
lic reflexes

(Q.70) If end-diastolic pressure is held constant, increasing which of the following will
increase the cardiac index?
(a)
Peripheral vascular resistance
(b)
Pulmonary wedge pressure
(c)
Heart rate
(d)
Systemic diastolic pressure
Your Response :
Correct Answer
C
:
Exp:
Heart rate (Schwartz, 7/e, p 849.)
The cardiac index is computed by dividing the cardiac output by
the body surface area; the cardiac output is the product of the stroke
volume and the heart rate [CI = CO/BSA; CO = SV HR;
therefore, CI = (SV HR)/BSA].
An increased heart rate will directly increase the cardiac output and
cardiac index.
The remaining choices in the question will either decrease or not
affect the stroke volume and consequently will not increase the
cardiac index.
(Q.71 Potency of anaesthetic agent can be estimated by:
Molecular weight
(a)
)
Lipid solubility
(b)
Minimal alveolar concentration
(c)
Boiling point
(d)
Your Response :
Correct Answer
C
:
Exp Minimal alveolar concentration
:
Potency of inhaled anaesthetic agents is estimated by the minimum alveolar concentration (MAC)
that produces a lack of reflex response to skin incision in 50% individuals.

Important Characteristics of Inhalational Agents


ANESTHETI POTENC SPEED OF SUITABILITY SENSITIZATION
C
Y
INDUCTION FOR
TO
AND
INHALATIONA CATECHOLAMIN
EMERGENC L INDUCTION ES
E
Nitrous oxide Weak
Fast
Insufficient alone None
Diethyl ether Potent
Very slow
Suitable
None
Halothane
Potent
Medium
Suitable
High
Enflurane
Potent
Medium
Not suitable
Medium
Isoflurane
Potent
Medium
Not suitable
Minimal
Sevoflurane Potent
Rapid
Suitable
Minimal
Desflurane
Potent
Rapid
Not suitable
Minimal

METABOLIZE
D (%)

Minimal
10
20+
<10
<2
<5
0.02

(Q.72) Commonest post operative complication of intubation is:


Sore throat
(a)
Aspiration pneumonitis
(b)
Vocal cord granuloma
(c)
Tracheal stenosis
(d)
Your Response :
Correct Answer
A
:
Sore throat
Exp:
Most common post operative complication of intubation is sore throat which usually
subsides in 2-3 days without any specific treatment.

(Q.73) Short half life of thiopentone is due to:


Excretion
(a)
Metabolism
(b)
Spontaneous degradation
(c)
Redistribution.
(d)
Your Response :
Correct Answer
D
:
Redistribution.
Exp:
The elimination half life of thiopentone is 10.4 hours but consciousness is regained
after 10-15 minutes due to redistribution which means drug from highly vascular
areas like brain is redistributed to less vascular areas like fat and muscle.

(Q.74) When should ticlopidine be stopped in a patient posted for inguinal hernia surgery?
Can be continued
(a)
Stop 3 days prior to surgery
(b)
Stop 7 days prior to surgery
(c)
Stop 14 days prior to surgery
(d)
Your Response :
Correct Answer
D
:
Stop 14 days prior to surgery
Exp:

Ticlopidineis anantiplatelet drugin thethienopyridinefamily. Likeclopidogrel, it

is anADP receptor inhibitor.


It is used in patients in whomaspirinis not tolerated, or in whom dual antiplatelet
therapy is desirable.
Because it has been reported to increase the risk ofthrombotic thrombocytopenic
purpura(TTP) andneutropenia, its use has largely been supplanted by the newer
drug,clopidogrel, which is felt to have a much lower hematologic risk.
The usual dose is 250 mg twice daily by the oral route. Ticlopidine's full
antiplatelet activity continues for 8 to 11 days after the first dose, and platelet
function is normalized approximately 14 days after the last dose.
Stop 10 to 14 days before surgery. Stop 14 days before Neuraxial Anesthesia
like spinal or epidural blocks in lower abdominal or lower limb surgeries.
Prolonged bleeding time is normalized within 2 hours after administration of
20mg methylprednisolone IV.
Platelet transfusions may also be used to reverse the effect of Ticlopidine on
bleeding. Because platelet transfusions may accelerate thrombosis in patients
with thrombotic thrombocytopenia on Ticlopidine, they should, if possible, be
avoided

(Q. A patient with history of myocardial ischemia is to be operated for


75) cholecystectomy, the anaesthetist would like to use:
(a)
Isoflurane
(b)
Enflurane
(c)
Desflurane
(d)
Halothane
Your Response :
Correct Answer
C
:
Ex Desflurane
p:
Isoflurane can cause coronary steal in myocardial ischemia so can not be used for
these patients. Halothane and enflurane reduces the cardiac output significantly
which can be detrimental in these patients. So the best technique is to maintain
anaesthesia on opioids especially morphine. Desflurane < 1 MAC can be used but
above 1 MAC it stimulates sympathetic system and can cause tachycardia.
Also remember : Cardiopulmonary Effects of Inhalational Anesthetics
INHALAT BLOO HEAR CARD SENSITIZAT VENTILA BRONCHODI
D
T
IAC ION TO
TORY
LATION
IONAL
PRESS RATE OUTP CATECHOL DEPRESS
AGENT
URE
UT
AMINES
ION
Nitrous oxide Little Little Little No
Minimal No
effect effect effect
Halothane
Marked Moderat Marke Marked
Moderate Moderate
dose- e
d dosedosedepend decreas depend
dependent
ent
e
ent
effect
decreas
decrea
e
se
Enflurane
Marked Moderat Moder Moderate
Marked
Minimal
dose- e
ate
dosedepend decreas dosedependent

ent
e
decreas
e
Isoflurane

Sevoflurane

Desflurane

depend
ent
decrea
se
Modera Variabl Minim Minimal
te dose- e
al
depend increase decrea
ent
se
decreas
e
Modera Little Moder Minimal
te dose- effect ate
depend
doseent
depend
decreas
ent
e
decrea
se
Minima Variabl Minim Minimal
l
e;
al
decreas marked decrea
e
increase se
with
rapid
increase
in
concent
ration

effect

Marked
Moderate
dosedependent
effect

Moderate Moderate
dosedependent
effect

Marked
Moderate
dosedependent
effect

(Q.76) Bag and mask ventilation is contraindicated in all, except:


(a)
Tracheoesophageal fistula with esophageal atresia
(b)
Meconium aspiration syndrome
(c)
Diaphragmatic hernia
(d)
Lung malformation
Your Response :
Correct Answer
D
:
Lung malformation
Exp:
Bag and mask ventilation is contraindicated in:
Full stomach patients.
Patients at high risk of aspiration like pregnancy and hiatus hernia.
Intestinal obstruction.
Diaphragmatic hernia (as there will be distension of bowel increasing further
respiratory difficulty).
TOF (air through fistula can lead to increase in gastric pressure).

Meconium aspiration syndrome (as patient has already aspirated bag and mask
ventilation will further increase aspiration).

(Q.77) Anaesthesia of choice for rigid bronchoscopy:


Topical
(a)
General anaesthesia with intubation
(b)
General anaesthesia with high frequency ventilation
(c)
Insufflation
(d)
Your Response :
Correct Answer
C
:
General anaesthesia with high frequency ventilation
Exp:
Rigid bronchoscopy is usually performed for removal of foreign body. The
bronchoscope occupies the trachea so intubation is not possible thereby patient is
ventilated through a small port of bronchoscope by high frequency jet ventilation.
Again apnea and insufflation methods are not preferred.

(Q.78) Anaesthesia of choice for draining peritonsillar abscess:


Local
(a)
General anaesthesia with ketamine
(b)
General anaesthesia with propofol
(c)
General anaesthesia with thiopentone
(d)
Your Response :
Correct Answer
A
:
Local
Exp:
Peritonsillar abscess is best drained under local anaesthesia. General anaesthesia
should be avoided as intubation is very difficult due to trismus and if the abscess is
opened inside the patient can aspirate.

(Q.79) Spinal anaesthesia is preferred in lower abdominal surgeries because it:


Gives deep analgesia
(a)
Gives good relaxation of abdominal muscles
(b)
Shrinks intestines so that other viscera are seen
(c)
Patient is conscious & cooperative
(d)
Your Response :
Correct Answer
C
:
Shrinks intestines so that other viscera are seen
Exp:
In spinal anaesthesia there is sympathetic block leading to parasympathetic
overdominance which causes contracted gut enabling a better view of structures of
lower abdomen like ovaries and uterus.

(Q.80) The accidental aspiration of gastric contents into the tracheobronchial tree should be initially treated
by

(a)
(b)
(c)

Tracheal intubation and suctioning


Steroids
Intravenous fluid bolus

Cricothyroidotomy
(d)
Your Response :
Correct Answer
A
:
Tracheal intubation and suctioning
Exp:
Gastric aspiration is best treated by tracheal suctioning, oxygen, and positivepressure ventilation.
Bronchoscopy is helpful if particulate matter is causing bronchial obstruction or if the
vomitus is found to contain particulate material.
Bronchial lavage is no longer recommended, and steroids have not been shown to be
of value. Fluids should be given sparingly because hypervolemia will worsen the risk
of pulmonary edema following aspiration.
Tracheostomy may be indicated for long-term airway management in obtunded or
otherwise severely debilitated patients; however, initial control of the airway should
be by orotracheal intubation whenever possible.
High positive end expiratory pressure is not required unless respiratory failure
develops.

(Q.81) Method of choice for sterilization of endotracheal tube is


Gamma rays
(a)
Boiling
(b)
Autoclaving
(c)
ETO gas
(d)
Your Response :
Correct Answer
D
:
ETO gas
Exp:
All disposable items like face mask, endotracheal tubes, airways are best sterilized by
ethylene oxide gas sterilization.

(Q.82) The drug for OPD analgesia is:


Morphine
(a)
Pethidine
(b)
Fentanyl
(c)
Alfentanil
(d)
Your Response
:
Correct Answer
D
:
Alfentanil
Exp:
Alfentanil is opioid of choice for day care (OPD) analgesia.

(Q.83) The cuff of tracheostomy tube should be:


High pressure, high volume
(a)
High pressure, low volume
(b)

Low pressure, low volume


(c)
Low pressure, high volume
(d)
Your Response :
Correct Answer
D
:
Low pressure, high volume
Exp:
The cuff of tracheostomy should be high volume and low pressure to
prevent tracheal ischemia. The cuff pressure should be maintained below
15 mmHg.

(Q.84) Most common cause of metabolic acidosis in. anaesthesia is:


Hypoventilation
(a)
Hypotension
(b)
Renal failure
(c)
Ketoacidosis
(d)
Your Response :
Correct Answer
B
:
Hypotension
Exp:
Hypotension (shock) causes tissue hypoxia leading to production of lactic acid and
this lactic acidosis is th6 most common cause of metabolic acidosis seen during
anaesthesia.

(Q.85) According to which law, the volume of gases is inversely proportional to pressure, temperature I
remaining constant:

Boyle's law
(a)
Charle's law
(b)
(c)
Dalton's law
Graham's law
(d)
Your Response :
Correct Answer
A
:
Boyle's law
Exp:
Boyle's law states that at constant temperature volume is inversely
proportional to pressure whereas as per Charle's law if pressure remains
constant then volume is directly proportional to temperature.

(Q.86) Lack system is modification of


Type A
(a)
Type D
(b)
Bain
(c)
Type F
(d)
Your Response :
Correct Answer
A
:
Type A
Exp:

Lack system is modification of type A (Magill) circuit. In this modification a


inner tube is incorporated in Magill to make it coaxial circuit. Here inner tube is used
for expiration and outer tube for inspiration (contrary to Bain circuit where inner tube
is used for inspiration and outer tube for expiration).

(Q.87) Artificial nose is:


Heat and moisture exchanger
(a)
Nose created from cartilage
(b)
Ultrasonic nebulizer
(c)
Ventimask
(d)
Your Response :
Correct Answer
A
:
Exp:
Heat and moisture exchanger
Heat and moisture exchanger is also called as artificial nose
and condenser humidifier. It is disposable instrument placed
between breathing circuit and endotracheal tube. It contains a
hygroscopic layer which retains the heat and moisture of expired
gases and deliver it in next breath to inspired gases.
(Q.88) Which of the following is contraindicated in renal failure?
(a)
Morphine
(b)
Pethidine
(c)
Fentanyl
(d)
Atracurium
Your Response :
Correct Answer
A
:
Exp:
Morphine
Ref. KDT Pharmacology
Morphine should not be used in renal failure because morphine
forms active metabolite morphine 6-glucoronide which is
excreted via kidney. It can accumulate and cause depression of
ventilation.
(Q.89) End tidal carbon dioxide is decreased in all except:
(a)
Pulmonary embolism
(b)
Cardiac arrest
(c)
Exhausted soda lime
(d)
Displacement of endotracheal tube
Your Response :
Correct Answer
C
:
Exhausted sodalime
Exp:

End tidal carbon dioxide is decreased in:


a. Pulmonary embolism.

b. Cardiac arrest (becomes zero).

c. Accidental extubation (becomes zero).


endotracheal tube.

d. Obstruction or disconnection of

It may increase in:


a. Exhausted sodalime.
b. Defective valves of closed circuit.
hyperthermia, thyrotoxicosis, fever.

c. Malignant

(Q.90) If malignant hyperthermia is suspected intraoperatively


Complete the procedure but pretreat with dantrolene prior to future elective surgery
(a)
Administer inhalational anesthetic agents
(b)
Administer succinylcholine
(c)
Hyperventilate with 100% oxygen
(d)
Your Response :
Correct Answer
D
:
Hyperventilate with 100% oxygen
Exp:
The cause of malignant hyperthermia is unknown, but it is associated with
inhalational anesthetic agents and succinylcholine.
It may develop in an otherwise healthy person who has tolerated previous surgery
without incident. It should be suspected in the presence of a history of unexplained
fever, muscle or connective tissue disorder, or a positive family history (evidence
suggests an autosomal dominant inheritance pattern).
In addition to fever during anesthesia, the syndrome includes tachycardia, increased
O2 consumption, increased CO2 production, increased serum K, myoglobinuria,
and acidosis. Rigidity rather than relaxation following succinylcholine injection may
be the first clue to its presence.
Treatment of malignant hyperthermia should include prompt conclusion of the
operative procedure and cessation of anesthesia, hyperventilation with 100% oxygen,
and administration of intravenous dantrolene.
The urine should be alkalinized to protect the kidneys from myoglobin precipitation.
If reoperation is necessary, one should premedicate heavily, alkalinize the urine, and
avoid depolarizing agents such as succinylcholine. Pretreatment for 24 h with
dantrolene is helpful; it is thought to act directly on muscle fiber to attenuate calcium
release.

(Q.91) Brain mask is:


Laryngeal mask airway
(a)
Type D Mapleson circuit
(b)
Face mask
(c)
Neonatal face mask
(d)
Your Response :
Correct Answer
A
:

Exp:

Laryngeal mask airway


Laryngeal mask airway (LMA) was discovered by Archies Brain, so called as Brain's
mask.

(Q.92) Most commonly used laryngoscope is:


Miller
(a)
Macintosh
(b)
McCoy
(c)
Oxford
(d)
Your Response :
Correct Answer
B
:
Macintosh
Exp:
Macintosh is most commonly used laryngoscope. It has curved blade and is available
in 4 different blade sizes.

(Q.93) Successful cardiac massage can produce:


30% of normal cardiac output
(a)
50% of normal cardiac output
(b)
75% of normal cardiac output
(c)
100% of normal cardiac output
(d)
Your Response :
Correct Answer
A
:
30% of normal cardiac output
Exp:
Cardiac massage can only produce 30% of normal cardiac output so restoration of
normal rhythm should be achieved as early as possible.

(Q.94) True about Dexmedetomidine is all except:


Is the dextrorotatory S-enantiomerofmedetomidine.
(a)
Likeclonidine, it is anagonistofalpha-2 adrenergic receptors
(b)
(c)
Compared tomidazolam,causing significantrespiratory depression
Dexmedetomidine may also offer a new paradigm in the
(d)
pharmacologic treatment of symptoms of distress (intractable pain,
agitation or delirium) at the end of life.
Your Response :
Correct Answer
C.
:
Exp:
Compared tomidazolam,causing significantrespiratory
depression
Compared tomidazolam, dexmedetomidine was similarly effective
for sedation, but shortened the time toextubation. It was associated
with lessdelirium,tachycardiaandhypertension. but more
bradycardia.
It also seemed to be superior to lorazepam forventilatedpatients in

theintensive care unit.[3]Compared to midazolam, dexmedetomidine


is superior due to reduced intensive care costs. The reduced costs
are due to a reduction inintensive care unitstay as well as
reducedmechanical ventilation.
Dexmedetomidine has sedative,analgesic,sympatholytic,
andanxiolyticeffects that blunt many of the cardiovascular
responses in theperioperativeperiod.
It reduces the requirements forvolatile anesthetics, sedatives and
analgesics without causing significantrespiratory depression.
Dexmedetomidine may be useful for the treatment of the
deleterious cardiovascular effects of acute intoxication
andoverdose.
Intravenous infusion of dexmedetomidine is commonly initiated
with a 1 mcg/kg loading dose, administered over 10 minutes,
followed by a maintenance infusion of 0.21.0 mcg/kg/hour.
(Q.95) Rapid sequence intubation (RSI) is done to prevent aspiration of gastric content, which of the
following is not done during RSI?
CRICOID PRESSURE is applied
(a)
Succinylcholine is used as muscle relaxant
(b)
Positive pressure ventilation done by bag and mask
(c)
Preoxygenation is mandatory
(d)
Your Response :
Correct Answer
C
:
Positive pressure ventilation done by bag and mask
Exp:

Rapid sequence induction


Indications
During emergency anaesthesia, aspiration of stomach contents
is a potential risk in all patients with an incompetent larynx.
Preoperative fasting and prokinetic agents reduce this risk but are
not always appropriate or available before essential emergency
surgery where there are time constraints on patient optimization.
Passing a nasogastric tube can allow removal of some of the
stomach contents.
Factors associated with a high risk of aspiration include:
Abdominal pathology, especially obstruction or ileus;
Delayed gastric emptying (e.g. pain, trauma, opioids, alcohol,
vagotomy)

Incompetent lower oesophageal sphincter, hiatus hernia, gastrooesophageal reflux disease;


Altered conscious level resulting in impaired laryngeal reflexes;
Neurological or neuromuscular disease;
Pregnancy;
Difficult airway;
The risk of aspiration in these patients is present throughout the
perioperative period, especially during induction and emergence
from anaesthesia.
Essential features of rapid sequence induction
Pre-oxygenation with 100% oxygen
Predetermined induction doses of drugs
Cricoid pressure
Cuffed endotracheal tube
Equipment and strategy to manage failed intubation

(Q.96) The following modes of ventilation may be used for weaning off patients from
mechanical ventilation except:
(a)
Controlled Mechanical ventilation (CMV).
(b)
Synchronized intermittent mandatory ventilation (SIMV).
(c)
Pressure support ventilation (PSV).
(d)
Assist - control ventilation (ACV).
Your Response :
Correct Answer
A
:
Exp:
Except CMV all are weaning mode as they are required to detach
the patient from ventilation.
(Q.97) Maximum decrease in vital capacity is seen in which position
Trendelenburg
(a)
Lithotomy
(b)
Rt. Lateral
(c)
Prone
(d)
Your Response
:
Correct Answer
B
:

Exp:

Lithotomy

Lithotomy position decreases the vital capacity most; around 1820%


(Q.98) What is latest monitor used for monitoring depth of anaesthesia?
BIS
(a)
Entropy
(b)
Evoked potential
(c)
EMG
(d)
Your Response :
Correct Answer
B
:
ENTROPY
Exp:
ENTROPY MONITORING- is a relatively new method of assessing
anaesthetic depth. It was commercially developed by Datex-Ohmeda, now part
of GE Healthcare IT It relies on a method of assessing the degree of
irregularity in electroencephalogram (EEG) signals. The founding principle
behind this theory is that the irregularity within an EEG signal decreases with
increasing brain levels of anaesthetic drugs. If we relate the irregularity to the
entropy within the signal, then an entropy scale can be assigned.
The signal is captured via a forehead mounted sensor, in a similar way employed
by bispectral index (BIS).
Entropy monitors produce two numbers (RE - Response Entropy, SE- State
Entropy) that are related to frequency bandpass used. Response Entropy
incorporates higher frequency components that include that of electromyogram
activity. The reason for using higher frequency bandpass in response entropy is
to allow faster response from the monitor in relation to clinical state.
Published studies show that entropy scores do relate to clinical levels of
anaesthetic depth. Most anaesthetic drugs are detectable by entropy monitoring, a
notable exception being nitrous oxide, in common with BIS monitoring. Future
studies may show reduced levels of intraoperative awareness when using this
type of monitoring. Future studies may also look into possibilities of influence
on outcome when using depth of anaesthesia monitoring.

(Q.99) During surgery for aortic arch aneurysm under deep hypothermic circulatory
arrest, which of the following anaesthetic agent administered prior to
circulatory arrest that also provides cerebral protection?
(a)
Etomidate
(b)
Thiopental sodium
(c)
Propofol
(d)
Ketamine
Your Response :
Correct Answer
B
:
Exp:
Thiopental sodium is the best agent for cerebral protection as the
decrease be energy requirement for brain.
(Q.100) All are true for Remifentanil except:Short acting agent
(a)
Metabolized by non specific esterase
(b)
Opioid of choice for renal patients
(c)

Can be used through spinal/epidural route


(d)
Your Response
:
Correct Answer
D
:
Can be used through spinal/epidural route
Exp:

It should not be used for analgesia through spinal/epidural route as


it contains glycine which can cause motor weakness.
(Q.101) Which of the following is bright on T1 Weighted MRI images?
Acute hemorrhage
(a)
Subacute hemorrhage
(b)
Chronic hemorrhage
(c)
Remote Hemorrhage with Encephalomalacia
(d)
Your Response
:
Correct Answer
B
:
Subacute hemorhage
Exp:
Ref. MRI by Scott Atlas

MRI Evaluvation of hemorrhages


Biochemical form Stage of
Time
T1
T2
hemorrhage
Oxyhemoglobin in Hyperacute Immediate Isointense Hyperintense
RBCs
to first
few hrs
Deoxyhemoglobin Acute
Hrs to
Isointense to Hypointense
in RBCs
days
hypointense
Methoxyhemoglobin Early
First
Hyperintense Hypointense
in RBCs
subacute several
days
Extracelullar metHb Late
Days to Hyperintense Hyperintense
subacute months
Ferritin and
Days to Iso to
Hypointense
Chronic
hemosedrin
infinite
hypointense
time
(Q.102) Most common radiological abnormality predisposing to developmental dysplasia of the hip?
Small femoral head epiphysis
(a)
Abnormal angle of femoral neck
(b)
Hypertrophied ligamentum teres
(c)
Developmental Acetabular dysplasia
(d)
Your Response
:
Correct Answer
D
:
Developmental Acetabular dysplasia (Ref. Baily and Love 24th ed. Fig.
Exp:

27.34)

DEVELOPMENTAL DYSPLASIA OF THE HIP (DDH)


This term is more accurate than the older congenital dislocation of
the hip (CDH) because it includes dislocation along with other less
severe forms, all of which have in common a dysplasia or natural
shallowness of the acetabulum. The femoral neck is usually
anteverted.
This is highest in the neonate where environmental factors such as
breech presentation, transient ligamentous laxity and the skills of
examiners can influence the rate of detection of instability.
A family history of DDH increases the risk up to 30 times.
The condition is five times more common in girls, possibly
related to hormonal factors producing joint laxity.
The left side is affected more commonly than the right.
Breech presentation, especially the extended position, firstborn
children and caesarean section are associated with increased
neonatal instability, possibly through restricted foetal movement.
A foot deformity or torticollis should also alert the examiner to
possible associated DDH.
Finally, there may be hip dislocation or dysplasia in association
with neuromuscular disease syndromic conditions and skeletal
dysplasia.
Ultrasonography is a valuable test. It is more sensitive than
clinical examination and can detect not only displacement of the
femoral head but also varying degrees of dysplasia.
(Q.103) What is the diagnosis?

(a)
(b)
(c)
(d)
Your Response
:
Correct Answer
:
Exp:

Optic nerve sheath meningioma


Optic nerve glioma
Caroticocavernous fistula
Cavernous hemangioma

A
Optic nerve sheath meningioma

Optic nerve sheath meningioma with classic tram tract


appearance on T1 weighted contrast enhanced MRI.
Optic nerve meningiomas account from approximately a third of all optic nerve
neoplasms; optic nerve gliomas are the most common entity. Unlikegliomas
which occur primarily in children, optic nerve meningiomas are usually seen in
adults (mean age at presentation 40 years), however up to 25% present in
children, in which case they tend to be more aggressive
Similar tomeningiomaselsewhere there is a female predilection
The vast majority of cases are sporadic, although patients
withneurofibromatosis type II(NF2) are at increased risk.

On axial or oblique sagittal imaging the enhancing tumour


surrounding the non-enhancingoptic nerveresults in the socalledtram-track sign. This is most evident in tumours with tubular
growth pattern.
(Q.104) Holman Miller sign on PNS CT scan?
Nasopharyngeal Ca
(a)
Juvenile nasopharyngeal angiofibroma
(b)
Carcinoma of ethmoidal sinus
(c)
Esthesioneuroblastoma
(d)
Your Response
:
Correct Answer B

:
Exp:

Juvenile nasopharyngeal angiofibroma

Juvenile nasopharyngeal angiofibroma:


Highly vascular usually benign tumor.
Usually arises from sphenopalatine foramen.
Exclusively seen in males, especially young boys, with 15-16
years being the most common age of presenation.
Preoperative tumor embolization can certainly prevent
intraoperative blood loss.
Multislice CECET is usual diagnostic modality.
Indentation on the posterior wall of maxiallary antrum is antral or
Holman Milller sign.
Surgical excision is Rx of choice.
(Q.105) Phantom tumor is:
(a)
Loculated pleural effusion
(b)
Sub-pulmonic effusion
(c)
Fissural effusion
(d)
Paracardiac effusion
Your Response
:
Correct Answer
C
:
Exp:
Fissural effusion
(Ref. Grainger & Allison's Diagnostic Radiology: A Textbook
of Medical Imaging, 4th Ed.pg. 327)
PHANTOM TUMOUR
Fissural interlobular loculation is seen particularly in heart failure and
may produce the so-called phantom tumour.
Viewed in lateral view it is sharply marginated and biconvex and has a tail
passing along the fissure.
The en face appearances depend on the thickness of the effusion.
A common problem in practice is the differentiation of encysted fluid in the
lower right oblique fissure from a middle lobe collapse. Observations that
favour a collapsed and consolidated middle lobe rather than an effusion include
non-homogeneity, a straight or concave border in the lateral view, a wedge-like

outline with the base reaching the sternum, and absence of the minor fissure.
In case of doubt, a CT scan can be performed.

(Q.106) Halo sign is characteristically seen in ?


(a)
Tuberculosis
(b)
Aspergillosis
(c)
Sarcoidosis
(d)
Silicosis
Your Response
:
Correct Answer
B
:
Exp:
Aspergillus
REF : Adam: Grainger & Allison's Diagnostic Radiology, 5th
ed. Chap 15
Classical named signs in Respiratory system Radiology :
SIGN / SPECIFIC
SEEN IN
FEATURE
Meniscus / Moon/ Air
Hydatid cyst of lung
crescent / Double arch sign
Cumbo sign
Water lilly / Camalotte sign
Serpent sign / Rising sun sign
Empty cyst sign
Popcorn calcification

Hamartoma

Westermark sign

Mediastinal nodes of
histoplasmosis
Pulmonary thrombo-embolism

Haptons hump
Palla sign
Fleishner lines
Felsons sign
Sail sign
Mulvay Wave sign

Thymic enlargement

Notch sign
Comet tail sign
Golden S sign
Luftsichel sign
Broncholobar sign
Ring around artery sign

Rounded atelectasis
RUL collapse secondary to a
central mass
LUL collapse
LLL collapse
Pneumo-mediastinum

Continuous diaphragm sign


Tubular artery sign
Double bronchial wall sign
V sign of Naclerio
Spinnaker sail sign
Deep sulcus sign

Pneumothorax

Visceral pleural line


Thumb sign
Steeple sign
Air crescent sign

Epiglottitis
Croup
Aspergilloma

Monod sign
Bulging fissure sign
Batwing sign
Collar sign

Klebsiella pneumonia
Pulmonary edema on CXR
Diaphragmatic rupture

Dependant viscera sign


Feeding vessel sign
Finger in glove sign
Halo sign
Head cheese sign
Juxtaphrenic peak sign
Reversed halo sign
Saber sheath trachea
Sandstorm lungs
Signet ring sign
Superior triangle sign
Split pleura sign
Tree in bud sign on HRCT

Pulmonary septic emboli


ABPA
Aspergillosis
Subacute hypersensitivity
pneumonitis
RUL atelectasis
Cryptogenic organized pneumonia
COPD
Alveolar microlithiasis
Bronchiectasis
RLL atelectasis
Empyema
Endobronchial spread in TB

(Q.107) Patient with head injury, unconscious, no mass / midline shift, CT shows basal
cistern collapse, multiple hemorrhage. Most likely cause?

(a)
(b)
(c)
(d)
Your Response
:
Correct Answer
:
Exp:

Diffuse axonal injury


Multiple cerebral infarcts
Concussion injury
Cerebral laceration

A
Diffuse axonal injury
Diffuse axonal injury (DAI)
It is caused by shearing of the white matter, often at the graywhite junction.
This is thought to be due to the differing tissue density or fixation
between two structures in differing response to rotation,
acceleration, and deceleration.
Detection is often associated with changes in the lobar white
matter, brainstem, and corpus callosum with ovoid or elongated
regions of decreased density.
Patients usually present with severe impairment of consciousness
from the moment of impact. MRI (FLAIR or T2 weighted) is
most useful in defining the extent of axonal shearing and non
hemorrhagic injury.
CT results are often negative, but acute areas of petechial
hemorrhage and cerebral edema have been seen in early stages.

(Q.108) Thyroid malignancy can not be diagnosed by


(a)
Hypo echogenicity
(b)
Hyper echogenicity
(c)
Microcalcifications
(d)
Irregular margins
Your Response
:
Correct Answer
B
:
Hyper Echogenicity
Exp:
Thyroid nodules
They are common and occur in up to 50% of the adult population; however, less
than 7% of thyroid nodules are malignant.
High-resolution ultrasonography (US) is commonly used to evaluate the thyroid
gland, and features that distinguish benign from malignant nodules include:

Microcalcifications are one of the most specific US findings of a thyroid


malignancy.
Other useful US features include a marked hypo echogenicity, irregular margins,
and the absence of a hypoechoic halo around the nodule.
Lymphadenopathy and local invasion of adjacent structures are highly specific
features of thyroid malignancy but are less commonly seen.

(Q.109) A radiological feature consistent with osteogenesis imperfecta:


Diffuse Osteopenia
(a)
Diffuse Osteosclerosis
(b)
Cotton wool lesions
(c)
Marble bone appearance
(d)
Your Response
:
Correct Answer
A
:
Diffuse Osteopenia
Exp:
Ref. J. Maheshwari 2nd ed. 267; above q for explanation
OSTEOGENESIS IMPERFECTA (OGI)

It occurs due to defect/mutation in genes responsible for type 1 collagen.


It is relatively rare disorder manifested by increased fragility of bones and
osteoporosis, dental abnormalities, wormian bones, and lax joints.
In osteogenesis imperfecta congenita osteoporosis occurs with short, broad and weak
long bones, while in osteogenesis imperfecta tarda, which is a benign form, the bones are
bowed, thin and gracile.

(Q.110) Corner sign of park is feature of:


Scurvy
(a)
Rickets
(b)
Battered baby syndrome
(c)
Sickle cell disease
(d)
Your Response
:
Correct Answer
A
:
Scurvy
Exp:
Ref. Sutton 7th ed. 1384
Scurvy-Radiographic findings:

a. Osteoporosis
b. Pensil thin cortex
sign d. Trumerfield zone of rarefaction
e. Corner sign of Park

c. Wimburgers

f. Line of Frankel

g.

Subperiosteal hemorrhage
(Q.111) Absent lateral third of clavicle is seen in:
Hypoparathyroidism
(a)
Turners syndrome
(b)
Fibrous dysplasia
(c)
Cleidocranial dysostosis
(d)
Your Response
:
Correct Answer
D
:
(Cleidocranial dysostosis)
Exp:
CLEIDOCRANIAL

DYSPLASIA/DYSOSTOSIS (CCD)

Autosomal recessive disease with faulty intramembranous bone ossification


Large head with small face, abnormal dentition and drooping hypermobile
shoulders
Radiological features:
Skull
a. Platybasia

b. Large foramen magnum

d. Persistent metopic suture


Brachycephaly

e. Hot cross bun skull

c. Wormian bones
f.

Thorax
a. Agenesis or hypoplasia of clavicles

b. Small winged scapulae

Pelvis
a. Underdeveloped pelvis

b. Coxa valgus early

Spine
a. Biconvex vertebrae

b. Hemi vertebrae

Extremities
Elongated 2nd metacarpal
D/d: pyknodysostosis (confused with CCD because of similar clavicular and skull
changes, and pyknodysostosis is also confused with osteopetrosis because generalized
increase in bone density, however these patients are not short which differentiate it from
CCD)

(Q.112) True about radiological features of Neurofibromatosis type 1 is all except:


Dural ectasias
(a)
Empty orbit sign
(b)

(c)
(d)
Your Response
:
Correct Answer
:
Exp:

Encephalotrigeminal angiomatosis
Superior rib notching

C
Encephalotrigeminal angiomatosis
Ref. Grainger diagnostic radiology 4th ed. 1980

Encephalotrigeminal angiomatosis is synonymous with Sturge Weber


syndrome, which is characterized by venous angiomas, Port wine stain and
tramtrack gyral calcification in cerebrum.

(Q.113) Judet view of X-ray is for ?


Pelvis
(a)
Calcaneum
(b)
Scaphoid
(c)
Shoulder
(d)
Your Response
:
Correct Answer
A.
:
Pelvis
Exp:
Ref : Adam: Grainger & Allison's Diagnostic Radiology, 5th ed. Chap 46
The routine radiographic assessment of the pelvis always begins with a standard
anteroposterior (AP) view and can be augmented with a variety of oblique views. Inlet
and outlet views, obtained by angling the tube caudally and cranially respectively, can
help in visualizing the obturator rings and assessing the integrity of the pelvic ring.
Judets views, obtained by rolling the patient to one obliquity or the other, can be
valuable in assessing the anterior or posterior walls of the acetabula.
CT is a more precise method of evaluating the acutely injured pelvis, allowing detection
and characterization of subtle fractures and fragments in areas of complex anatomy.
In addition, it provides detailed information about the soft tissues in and around the
pelvis, such as haematomas. With post-processing techniques, three-dimensional
reconstructions of the traumatized pelvis can be created and manipulated to provide the
orthopaedic surgeon with information useful in planning treatment
Judet's views are standard radiographic projections which are employed in patients with
acetabulum fractures. Judet's views are generally only performed as a supplementary
view. In cases of acute injury, they can be useful in demonstrating or confirming
acetabular fractures. In a clinic referral situation, Judet views may be requested to
follow-up fracture healing.
Judet views are basically 45 degree obliques of the affected hip. The 45 degree angle is
best achieved by rolling the patient. Alternatively, tube angulation (with careful
consideration of grid line orientation) is a legitimate alternative. In patients with acute
injuries, rolling the patient will be very painful.
Consultation directly with the referring doctor (or preferably the orthopaedic surgeon)
will ensure that your approach to performing these views is safe for the patient.If the

Judet views are performed using a bedside technique, a non-grid technique will ensure
that the image is not marred by grid cut-off. The image quality which can be achieved
with this technique is limited due to excessive scatter radiation. Transfer of the patient
onto the X-ray table should be performed with the consent of the referring doctor (and
the patient).
This is just ONE named Radiographic projection. This question necessitates that we go
through few of the important Eponymous radiographic projections. Below is a overexhaustive list of such projections. I agree it is practically impossible to remember all of
them, yet do go through it atleast once and try and remember those in bold font.

(Q.114) Which of the following is a X ray feature of fibrous dysplasia?


Ground glass appearance
(a)
Calcification
(b)
Cortical thickening
(c)
Well defined borders
(d)
Your Response
:
Correct Answer
A
:
Ground glass appearance
Exp:
Radiographic features of fibrous dysphasia:
The typical radiographic appearance of fibrous dysplasia consists of:
A medullary-based, minimally expansile lesion with ground-glass opacity and
irregular but well-defined borders.
In long bones, the location usually is diaphyseal or diametaphyseal, and the epicenter is
centric or eccentric.
More expansile lesions cause endosteal scalloping and thinning that weakens the cortex.
Lesional radiopacity is variable depending on the ratio of fibrous and osseous tissue.
Homogeneous, featureless grey opacity is the classic ground-glass appearance of
fibrous dysplasia, a term borrowed from the appearance of frosted or ground window
glass that is uniformly opaque. Lesions are less commonly homogeneously lucent or
sclerotic.
Chronic changes secondary to bone weakness may lead to bowing of weight-bearing
structures, fracture, and remodeling. The shepherds crook is the bowing deformity
with varus angulation of the proximal femur.
Vertebral bodies in the polyostotic form of fibrous dysplasia have mixed density lesions.
The early radiological features of sarcomatous transformation are moth-eaten or cystic
areas of osteolysis within a lesion.
Cortical destruction and insidious formation of a soft-tissue mass containing tumor
matrix are characteristic of underlying osteosarcoma.
Ring-like and spotty calcification of cartilaginous matrix is indicative of
chondrosarcoma. Fibrosarcoma has no visible matrix and shows simple osteolysis

(Q.115) The best investigation for temporal bone injuries


CT scan
(a)

(b)
(c)
(d)
Your Response
:
Correct Answer
:
Exp:

Angio
Plain film
MRI

A
CT scan
CT is the primary investigation of choice for clinically important brain injury
MRI is not currently indicated as the primary investigation
Skull x-rays have a role in the detection of non-accidental injuries in children
CT should be available within one hour of being requested
Indications for CT
GCS less than 13 at any point since the injury
GCS equal to 13 or 14 at 2 hours after the injury
Suspected open or depressed skull fracture
Any sign of basal skull fracture
Post-traumatic seizure
Focal neurological deficit
More than one episode of vomiting
Amnesia for greater than 30 minutes of events before impact
If LOC in patients older then 65 years, coagulopathy or dangerous mechanism of injury

(Q.116) Radiological Signs of Acute Pancreatitis include the following EXCEPT:


(a)
Colon cut-off sign
(b)
Murphy sign
(c)
Renal halo sign
(d)
Sentinel loop sign
Your Response
:
Correct Answer
B
:
Exp:
Murphy sign
Ref. Bailey and Love 24th ed. 1123; Sutton Radiology 7th ed. 686
RADIOGRAPHIC SIGN OF ACUTE PANCREATITIS

Gas filled dilated duodenal cap and loop


Sentinel loop sign (single dilated small bowel loop), optimally
demonstrated in left lateral decubitus view
Renal halo sign (the left kidney has surrounding halo due to
edema and is displaced down)
Colonic cut-off sign (dilated transverse colon becomes abruptly
gasless in the region of splenic flexure)
Gasless abdomen due to vomiting
Dilated loop of other adjacent bowel coils (small bowel, terminal
ileum, ascending and transverse colon), Paralytic ileus
Loss of left psoas outline
Absent right psoas shadow
Elevated left diaphragm
Left pleural effusion
(Q.117) Maximum radiation exposure is by:
(a)
Radiography
(b)
CT abdomen
(c)
Radionuclide scan
(d)
X-ray Abdomen
Your Response
:
Correct Answer
B
:
Exp:
CT abdomen
Ref. Bailey and Love 24th ed. Pg. 7, Box. 2.1
Examination
Effective total dose

Chest radiograph
Skull radiograph
Pelvis radiograph
Lumbar spine radiograph
Upper GI series (Barium)
Abdomen radiograph
Barium enema
IVP/IVU

(mSv)
0.06
0.2
0.65
1.3
2.45
0.55
2.8 to 4
1.6

Extremities
Enteroclysis
CT chest
CT abdomen
CT head
RNI

0.01
1.5
8
10
3.5
4.8

(Q.118) Minimum pneumothorax is best seen in which of the following view (chest x-ray)?
(a)
Lordotic view
(b)
Right lateral view
(c)
Left lateral view
(d)
Chest x-ray in complete expiration
Your Response
:
Correct Answer
D
:
Exp:
Chest x-ray in complete expiration
Ref. Sutton Radiology 7th ed.4
VARIOUS RADIOGRAPHIC VIEWS OF CHEST
Good visualization of the apices of lung require projection of
clavicles upward, as in the apical view (apicogram) with the Xray tube angled up 50-600, or downward, as in the lordotic view
with the patient in lordotic PA position in which a middle lobe
collapse is seen clearly as a well-defined triangular opacity. A
lordotic view is useful to detect lesions of middle lobe (like
collapse) and collection on in fissure and subtle infiltrates in UZs.
Oblique views are taken usually to demonstrate the retrocardiac
space, the posterior CP angles and the chest wall, with pleural
plaques being clearly demonstrated.
Decubitus view shows small amount of pleural fluid, which is not
seen on PA view.
Paired inspiratory and expiratory views are important in
demonstrating:
i. Air trapping
Interstitial shadowing

ii. Small pneumothorax

iv. Diaphragm movements


(Q.119) GFR is best measured with which of the following?
Iodohippurate
(a)
Tc99m-DTPA
(b)

iii.

v. Inhaled foreign body in children.

Tc99m-MAG3
(c)
Tc99m-DMSA
(d)
Your Response
:
Correct Answer
C
:
Exp:
Tc99m-MAG3

RENAL SCAN
I-131 01H (Orthoiodohippurate)

Largely replaced by Tc 99m MAG3 used for evaluation of Renal


tubular function / effective renal plasma flow.
Tc 99m DTPA ( Diethlene triamine Pentaacetic acid )

It is agent of choice for assessment of ;


i. Perfusion
ii. GFR
iii. Obstructive
uropathy
iv. Vesicouretral reflux
Tc 99m DMSA( dimercoptosuccinic acid )

It is suitable for imaging of functioning cortical mass


pseudotumor versus the lesion .
Tc 99m Mercaptoacetyltriglycine ( MAG3)

i. Replacing DTPA.
detected.

ii. True renal plasma flow

ACE inhibitor scintigraphy

It is for screening of Reno vascular hypertension.


(Q.120) Not a true statement about Osteoclastoma?
Usually solitary
(a)
Metaphyseal origin
(b)
Benign bone tumor
(c)
Common in age between 20-40 years
(d)
Your Response
:
Correct Answer
B
:
Exp:
Metaphyseal origin
Ref. Bailey and Love Surgery 24th ed. 435; Maheshwari
orthopaedics 2nd ed. 215

GIANT CELL TUMOR (OSTEOCLASTOMA)


It is an aggressive but benign tumor of bone.
It is uncommon before age 20 or after 50; 65% of our patients
were between ages 20 and 40 years.
Location: Giant cell tumors have a preference for the knee (50%).
Over 50% of giant cell tumors originate in the femur, tibia, or
fibula. Other site sof involvement -- 12% occur in distal radius and
6% in proximal humerus.
Tumors of the spine are unusual.
The most common symptom is pain and swelling about the joint
(usually the knee).
Giant cell tumors are epiphyseal in origin and they arises after
epiphyseal closure. When the lesion is first seen, it is already of
good size. Occasionally, it may present as a small lesion.
(Q.121) Shenton line is seen in X ray of ?
(a)
Knee
(b)
Shoulder
(c)
Elbow
(d)
Hip
Your Response
:
Correct Answer
D
:
Exp:
Hip
Ref : Canale & Beaty: Campbell's Operative Orthopaedics, 11th
ed. Chap 33
Shenton lineis an imaginary line drawn along the inferior border
of thesuperior pubic ramus (superior border of theobturator
foramen) and along the inferomedial border of the neck offemur.
This line should be continuous and smooth.
Interruption of the Shenton line can indicate (in the correct clinical
scenario)
developmental dysplasia of the hip (DDH)
fractured neck of femur

Also note the following important line as and planes pertaining to

hip radiology
Ilio-pectineal
line

On the standard AP view of the pelvis, the


iliopectineal line (also called the iliopubic line)
extends from the medial border of the iliac
wing, along the superior border of the superior
pubic ramus to end at the pubic symphysis .
This line is seen as the inner margin of the
pelvic ring and defines the anterior column of
the pelvis . The anterior column is well
demonstrated by a 45- degree anterior oblique
radiograph. Fractures extending through the
anterior column disrupt the contour of this line.
In addition, this line may be thickened in
patients with Paget disease7 or in patients with
familial idiopathic hyperphosphatasia.
Ilio-Ischial line The ilioischial line also begins at the medial
border of the iliac wing and extends along the
medial border of the ischium2-4 to end at the
ischial tuberosity. This defines the posterior
column of the pelvis. The posterior column is
well demonstrated by a 45-degree posterior
oblique radiograph. Fractures extending
through the posterior column of the pelvis
disrupt the contour of the ilioischial line
Theline of Kleindescribes a line along the
Line of Klein
superior edge of the neck of femur. It is useful
in detecting earlySUFEin adolescents. The line
should normally intersect the lateral part of the
superior femoral epiphysis. If theline of
kleinfails to intersect the epiphysis during the
acute phase, it is calledTrethowan's sign
Shentons line curve ensuring there is normal anatomic
relationship of the hip on the AP pelvis
radoigraph view. It is the arc formed by the
inferior surface of superior pubic ramus and the
medial surface of the proximal femur. If this
line is disrupted one should think about hip
fractures and a coxa valga abnormality.
Hilgenreiners line drawn on the AP pelvis radiograph
between the two Y-shaped triradiate cartilages
line
of both acetabulii. A perpendicular line is then
drawn down the anteriorinferior iliac spine.
Normally the femoral epiphysis should lie in
the lower inner quadrant. If there is
developmental hip dysplasia the femoral
epiphysis will lie in the superolateral quadrant.
A vertical line to Hilgenreiner's line through the
Perkins line
lateral rim of the acetabulum'
The Y line is a vertical line drawn from the
Y - line

superior most part of the femur to intersect with


a transverse line drawn through both triradiate
cartilages on the AP pelvis.
The acetabular angle/index is the slope of the
Acetabular
acetabular roof. It is the angle that lies between
angle / index
Hilgenreiner's line and a line drawn from the
most superolateral ossified edge of acetabulum
to the superloateral margin of the triradiate
cartilage.
The most commonly used method for
Caffeys line
measurement of the angle is Caffey's which
involves drawing a horizontal line joining the
superior margins of the radiolucent triradiate
cartilages (see Hilgenreiners line). A line is
then drawn from the superolateral margin of the
ossified acetabulum to the superolateral margin
of the triradiate cartilage. In the neonate the
angle of intersection is normally less than 30
and by 2 years of age the angle is normally less
than 20.
Sharps method n alternative method of assessing the acetabular
angle is Sharp's method which encompasses the
entire acetabulum as opposed to only the upper
(iliac) aspect. Sharp's method can be used over
the age of 9 months when the acetabular
teardrop becomes radiologically visible. A line
is drawn connecting the inferior points of the
acetabular teardrop bilaterally, and a second
line is then drawn connecting the superolateral
and inferomedial extremities of the acetabulum.
In infants the angle of intersection should be
less than 50 and gradually reduces to less than
38 in the adolescent. Developmental dysplasia
of the hip is associated with an increased
acetabular angle. Down's syndrome is
associated with a reduced acetabular angle.
Ultrasound is the preferred method of assessing
hip dysplasia.
The gluteal fat stripe is seen as a straight line
Gluteal fat
paralleling the superior aspect of the femoral
stripe
neck on a true AP radiograph and represents
normal fat between the gluteus minimus tendon
and the ischiofemoral ligament. This line
bulges superiorly in the presence of a hip joint
effusion
The iliopsoas fat stripe is seen as a lucent line
Ilio-psoas fat
immediately inferior to the iliopsoas tendon
stripe
The obturator fat stripe parallels the
Obturator fat
iliopectineal line and is formed by normal
stripe

pelvic fat adjacent to the obturator internus


muscle , which may be displaced by fracture,
hematoma, or mass.
(Q.122) Air crescent sign is seen in:
(a)
Hydatid cyst
(b)
Aspergillosis
(c)
Tuberculosis cavity
(d)
Amoebic liver abscess
Your Response
:
Correct Answer
B
:
Exp:
Aspergillosis
Air crescent sign is seen in fungal ball or aspegilloma in a tubercular cavity.
Radiographic and related imaging studies are important in detecting and
evaluating echinococcal cysts. Plain films will define pulmonary cysts of E.
granulosus usually as rounded masses of uniform density but may miss
cysts in other organs unless there is cyst wall calcification (as occurs in the
liver). MRI, CT, and ultrasound reveal well defined cysts with thick or thin
walls. When older cysts contain a layer of hydatid sand that is rich in
accumulated scolices, these imaging methods may detect this fluid layer of
defferent density Floating Water- Lily sign. However the most path
gnomonic finding, if demonstrable, is that of daughter cysts within the larger
cyst. This finding, like effshell or mural calcification on CT, is indicative of E.
granulosus infections and helps ot distinguish the cyst form carcinomas,
bacterial or amebic liver abscesses, or hemangiomas.

(Q.123) 'Sigmoid' esophagus on Ba Swallow is seen in:


Achalasia cardia
(a)
Scleroderma
(b)
Diffuse esophageal spasm
(c)
Carcinoma esophagus
(d)
Your Response
:
Correct Answer A
:
Achalasia cardia
Exp:
Ref. Sutton's textbook of radiology, 549
Achalasia cardia on Ba swallow may reveal mega or sigmoid esopahguw with bird beak
narrowing of cardiac end of esophagus.
Esophageal motility disorders like diffuse esophageal spasm, presbyesophagus and nutcracker
esophagus may show cork screw apperance.

(Q.124) Fallen fragment sign:


(a)
(b)
(c)
(d)

Chondroblastoma
unicameral bone cyst
Eosinophilic granuloma
Fibrous dysplasia

Your Response
:
Correct Answer
:
Exp:

B
unicameral bone cyst
Chondroblastoma
Unicameral bone cyst
fragment

: benign epiphyseal bone tumor.


: Proximal humerus is most common site and fallen or hidden

sign may be seen within.


Eosinophilic granuloma
Fibrous dysplasia

: Lytic lesion in skull with beveled edges


: Benign bone tumor with groundglass appearance of matrix.

(Q.125) In the diffuse axonal injury, the typical location of lesions in the brain include
Para-saggital region
(a)
Cerebral cortex
(b)
Ventricles
(c)
White matter of cerebral hemispheres, corpus callosum and the upper brain stem
(d)
Your Response
:
Correct Answer
D
:
Exp:
White matter of cerebral hemispheres, corpus callosum and
the upper brain stem
(Ref. Grainger & allison's diagnostic radiology 4th ed.2399)
Diffuse axonal head injury
This type of brain damage occurs as a result of mechanical shearing following
deceleration, causing disruption and tearing of axons, especially at the
grey/white matter interface.
Severity can vary from mild confusion to coma and even death.
Macroscopically, punctate haemorrhages are visible, especially in the corpus
callosum and superior cerebellar peduncle. Microscopically, retraction balls
reflecting axonal damage and microglial clusters (hypertrophied microglia) are
found diffusely in the white matter.
It is a frequent result of traumatic deceleration injuries and a frequent cause of
persistent vegetative state in patients.
Dai is the most significant cause of morbidity in patients with traumatic brain
injuries, which most commonly are the result of high-speed motor vehicle
accidents.
Typically, the process is diffuse and bilateral, involving the lobar white matter
at the gray-white matter interface.

The corpus callosum frequently is involved, as is the dorsolateral rostral


brainstem.
The most commonly involved area is the frontal and temporal white matter,
followed by
i. The posterior body and splenium of the corpus callosum,
caudate nuclei,
iii. Thalamus,

iv. Tegmentum,

ii. The

v. Internal capsule.

Internal capsule lesions are associated more frequently with hemorrhage than
are the other lesions and are secondary to the proximity of the lenticulostriate
vessels.

(Q.126) All are true about Left ventricular failure except:(a)


Pulmonary oligemia
(b)
Kerley B lines
(c)
Redistribution to apices
(d)
Cardiomegaly
Your Response
:
Correct Answer
A
:
Exp:
Pulmonary oligemia
Ref: Harrison, 17th edition, page 1447
Chest X-Ray
This provides useful information about cardiac size and shape, as
well as the state of the pulmonary vasculature, and may identify
noncardiac causes of the patient's symptoms.
Although patients with acute HF have evidence of pulmonary
hypertension, interstitial edema, and/or pulmonary edema, the
majority of patients with chronic HF do not.
The absence of these findings in patients with chronic HF reflects
the increased capacity of the lymphatics to remove interstitial
and/or pulmonary fluid.
In LVF there can be Kerley B lines are fine, dense, opaque,
horizontal lines that are most prominent in the lower and mid-lung
fields and that result from distention of interlobular septae and
lymphatics with edema when the resting mean LA pressure
exceeds approximately 20 mmHg.
In LVF upper lobe veins become prominent so called COLONEL

MOUSTHCHE Sign.
Cardiomegaly is also feature of heat failure.
Pulmonary oligemia is not a feature of heat failure it is seen in
right to left shunt. Like TOF.
(Q.127) A newborn has a posterior mediastinal cystic mass with several vertebral
anomalies. The most likely diagnosis would be which of the following?
Neuroblastoma
(a)
Bronchogenic cyst
(b)
Neuroenteric cyst
(c)
Myelocele
(d)
Your Response
:
Correct Answer
C
:
Exp:
Neuroenteric cyst
Ref. Sutton's textbook of radiology, 6th ed., 78 and 565
Neuroenteric cyst / esophageal duplication cyst is round or tubular
lesion occurring in the lower posterior mediastinum, which
often distort esophagus but only rarely communicate with the
esophageal lumen a chest x-ray may show a right-sided
mediastinal mass and associated vertebral anomalies like
hemivertebra, block vertebra, butterfly vertebra or spina bifida, if
present. Proximal esophageal duplication cysts may be associated
with tracheal compression and present with upper airway
obstruction while distal esophageal cysts are often asymptomatic
and found as an incidental chest x-ray finding. Barium meal will
confirm the presence of a smooth extrinsic esophageal filling
defect and CT/MRI will demonstrate its cystic nature.
Bronchogenic cysts are usually solitary asymptomatic mediastinal
masses which may present at any age. Typically they are thinwalled with a respiratory or enteric mucosal lining, which often
contains cartilage and mucous glands. The cyst contents usually
consist of thick mucoid material. The cysts can grow very large
without causing symptoms, but they may compress surrounding
structures.
(Q.128) Which of the following indicates radiograph contrast induced nephropathy?
(a)
Increased creatinine levels
(b)
Decreased urine output
(c)
Increased bilirubin
(d)
Decreased bilirubin
Your Response
:

Correct Answer
A
:
Exp:
Increased creatinine levels
REF :
1. http://emedicine.medscape.com/article/246751-overview : Contrast-Induced
Nephropathy Author: Renu Bansal, MD; Chief Editor: Vecihi Batuman,
2. American Journal of Roentgenology. 2004;183: 1673-1689.
10.2214/ajr.183.6.01831673
Contrast-induced nephropathy (CIN) is defined as the impairment of renal
function and is measured as either a 25% increase in serum creatinine (SCr)
from baseline or 0.5 mg/dL (44 mol/L) increase in absolute value, within 4872 hours of intravenous contrast administration.
For renal insufficiency (RI) to be attributable to contrast administration, it should be
acute, usually within 2-3 days, although it has been suggested that RI up to 7 days
postcontrast administration be considered CIN; it should also not be attributable to
any other identifiable cause of renal failure.
A temporal link is thus implied. Following contrast exposure, SCr levels peak
between 2 and 5 days and usually return to normal in 14 days.
Complications
CIN is one of the leading causes of hospital-acquired acute renal failure. It is
associated with a significantly higher risk of in-hospital and 1-year mortality, even
in patients who do not need dialysis.
Nonrenal complications include procedural cardiac complications (eg, Q-wave MI,
coronary artery bypass graft [CABG], hypotension, shock), vascular complications
(eg, femoral bleeding, hematoma, pseudoaneurysm, stroke), and systemic
complications (eg, acute respiratory distress syndrome [ARDS], pulmonary
embolism).
There is a complicated relationship between CIN, comorbidity, and mortality. Most
patients who develop CIN do not die from renal failure.
Death, if it does occur, is more commonly from either a preexisting nonrenal
complication or a procedural complication.
Other renal function markers
The use of SCr as a marker of renal function has its limitations. Indicators such as
the estimated glomerular filtration rate (eGFR) and cystatin C are increasingly
considered to be more reliable and accurate reflectors of existing renal function.
The eGFR can be calculated using the Modification of Diet in Renal Disease
(MDRD) formula or the Cockroft-Gault formula.
The Cockroft-Gault formula calculates eGFR using age, sex, and body weight,
which are factors that, independent of GFR, influence SCr. The MDRD equation
also includes blood urea nitrogen (BUN) and serum albumin.

The eGFR works best at low creatinine values. SCr and GFR share a curvilinear
relationship. At lower SCr values, doubling SCr is associated with a corresponding
50% decrease in GFR.
However, in elderly patients withchronic kidney disease(CKD) who have high
SCr values at baseline, a 25% rise in SCr is actually indicative of a relatively
modest reduction in GFR.
Nonetheless, even a 25% increase in SCr in this situation has been shown to have
great impact, especially in terms of inhospital and 1-year mortality.
Serum cystatin C is a serum protein that is secreted by nucleated cells. It is freely
filtered by the glomerulus and has been found to be an accurate marker of GFR.
Compared with SCr, cystatin C changes much earlier after contrast administration
and is not subject to confounding factors, such age, sex, and muscle mass, that
influence SCr values independent of the underlying GFR.
Cystatin C is increasingly being used as a marker of renal function in cardiac
surgical patients.

(Q.129) McConnells sign in echo-cardiography is seen in?


MI
(a)
Pulmonary embolism
(b)
Sarcoidosis
(c)
HOCM
(d)
Your Response
:
Correct Answer
B
:
Pulmonary embolism
Exp:
The specific appearance of the right ventricle on echocardiography is
referred to as the McConnell's sign. This is the finding of akinesia of the midfree wall but normal motion of the apex. This phenomenon has a 77%
sensitivity and a 94% specificity for the diagnosis of acute pulmonary
embolism
GRAINGERs DIAGNOSTIC RADIOLOGY 5th Ed Chapter 26 states :
The most common radiographic signs of pulmonary embolus (not causing
infarction) are:

Localized peripheral oligaemia (Westermark's sign)


secondary to the embolus lodging in a peripheral artery. It
may be associated with proximal arterial dilatation. Diagnosis
of this often subtle sign may be made easier by comparison
with previous radiographs (if available).
Peripheral airspace opacification, which represents pulmonary
haemorrhage.
Linear atelectasiscaused by ischaemic injury leading to
surfactant deficiency (as a result of damaged Type II
pneumocytes).
Pleural effusionoften small (in the absence of infarction).

Enlargement of the central pulmonary arteries may be found also,


but normally occurs secondary to chronic repeated embolic
disease. Less commonly cardiac enlargement may be seen in
patients with a large PE with associated cor pulmonale.
Infarction is relatively rare after embolism (occurring in < 10% patients). It is
most common in those patients with impaired cardiac function (causing
impairment to collateral flow). The changes seen in those without infarction
may be seen in patients with infarction also. To distinguish infarction from
noninfarction causing pulmonary embolus radiographically is tricky.
Radiographic signs more associated with infarction that may be of use
include:

A pleurally based, wedge-shaped opacity (Hampton's


hump), which is normally seen in the lateral or posterior
costophrenic sulcus of the lung. It may rarely contain air
bronchograms. The apex of the triangle points toward the
occluded feeding vessel and its base rests against the pleural
surface. An infarct is rarely truly wedge- or triangularshaped owing to the plane of the radiograph and surrounding
haemorrhage. Although often mentioned, it is neither specific
nor commonly seen in PE.
Consolidation (which may be multifocal) is seen predominantly in
the lower lobes. This may be seen from 12h up to several days
post embolism. Its exact shape may vary depending on the
location and state of the underlying lung
Distinction between parenchymal changes related to infarction
and those where no infarction has occurred may be made by
observing serial radiographs. Relatively rapid resolution of
parenchymal changes (up to 710d) is associated with
noninfarction-causing emboli, whereas those associated with
infarction cause changes that take considerably longer (weeks to
months) to resolve, and in addition normally heal with scarring or
localized pleural thickening.
Cavitation (which is reported to occur in < 5% of cases and is
normally related to secondary infection at the site of infarction or
due to a septic embolus).
Haemorrhagic pleural effusion (which may be slow to clear) and
may be seen in up to 50% of patients with PE. Such an effusion
may be the cause of ipsilateral chest pain
(Q.130) Clay Shovels fracture involves?
Spinous process
(a)
Lamina
(b)
Pedicle
(c)
Body
(d)
Your Response
:
Correct Answer
A
:
Exp:
Spinous process

Ref. Grainger & Allison's Diagnostic Radiology: A Textbook of


Medical Imaging, 4th Ed. 1823
Clay shovellers fracture is an avulsion of the spinous process
of C6, C7 or T1 caused either by rotation of the upper trunk when
the cervical spine is relatively fixed or, less commonly, as the
result of a direct blow.
(Q.131) Differentiating point between psoriatic and rheumatoid arthritis are the following EXCEPT?
Pencil-in-cup deformity
(a)
Telescoping of phalanges
(b)
Martels sign
(c)
Tufting of phalanges
(d)
Your Response
:
Correct Answer
C
:
Exp:
Martels sign
Ref. Harrison's Principles of Internal Medicine 16th Edition 1998
Martels sign is the overhanging edge sign of tophaceous gout.
PSORIATIC ARTHRITIS
The peripheral and axial arthropathies in psoriatic arthritis (PsA)
show a number of radiographic features that distinguish them from
RA and AS, respectively.
Characteristics of peripheral PsA include DIP involvement,
including the classic pencil-in-cup deformity; marginal erosions
with adjacent bony proliferation (whiskering); small joint
ankylosis; osteolysis of phalangeal and metacarpal bone, with
telescoping of digits; and periostitis and proliferative new bone at
sites of enthesitis.
Characteristics of axial PsA include asymmetric sacroiliitis;
compared with idiopathic AS, less zygoapophyseal joint arthritis,
fewer and less symmetric and delicate syndesmophytes; fluffy
hyperperiostosis on anterior vertebral bodies; severe cervical spine
involvement, with a tendency to atlantoaxial subluxation but
relative sparing of the thoracolumbar spine; and paravertebral
ossification.
Ultrasound and MRI both readily demonstrate enthesitis and
tendon sheath effusions that can be difficult to assess on physical
examination.

(Q.132) Sensitive most investigation to detect minimal intraperitoneal free air:


Standing x-ray abdomen
(a)
Right lateral decubitus x-ray
(b)
Chest x-ray
(c)
Left lateral decubitus x-ray
(d)
Your Response
:
Correct Answer
C
:
Chest x-ray
Exp:
Although as little as 1 ml of free gas can be demonstrated radiographically,
either in chest PA or a left lateral decubitus abdominal film, CT is superior to
plain radiographs in detecting minute quantities of pneumoperitoneum.
Chest PA view is more sensitive than even a left lateral decubitus abdominal
film.
CT is most sensitive investigation for detection of free peritoneal gas. Small
pockets of free air can be seen over liver and anteriorly in mid-abdomen or
peritoneal recesses.
In order not to miss small amounts of free gas, the images should be reviewed
on lung window setting.
Free air demonstrated in CT can be distinguished from bowel gas due to its
nondependent location and lack of haustra or valvulae.

(Q.133) Radiological Contrast to be preferred for demonstrating TO fistula:


High osmolar iodine contrast
(a)
Low osmolar iodine contrast
(b)
Gadolinium-DTPA
(c)
Thick barium sulphate suspension
(d)
Your Response
:
Correct Answer
B
:
Low osmolar iodine contrast
Exp:
Tracheoesophageal fistula
The above findings suggest a clinical diagnosis of tracheoesophageal fistula, which
usually occurs in association with esophageal atresia but may also occur as an isolated
anomaly.
Plain radiography may show air in esophagus and stomach (most common variety being
upper end of esophagus, being blind, and lower end connected to trachea).
Actually, contrast study of esophagus (with water soluble low-osmolarity nonionic
contrast agent) confirms the diagnosis.
If simple swallow doesn't show the fistula, modification may be required in the form of
contrast injection into ng tube with patient prone and as the tube (rubber) is withdrawn

the fistula is seen.

(Q.134) A suspected case of Dysphagia lusoria should be best investigated by:


HRCT
(a)
Spiral CT
(b)
MRA
(c)
DSA
(d)
Your Response
:
Correct Answer
C
:
MRA
Exp:
Dysphagia lusoria
Dysphagia due to vascular rings is known as dysphagia lusoria.
Abnormalities of the great vessels can contract the structures in mediastinum,
namely the trachea and esophagus.
Many vascular rings can be explained by the persistence of failure to regress of
parts of the aortic arch system during embryonic development.
Barium swallow, bronchoscopy, or angiographies are investigations to diagnose
but angiography is diagnostic.
Impressions and displacement of esophagus from vascular causes may be
congenital or acquired. Congenital lesions like aberrant right subclavian artery
(0.5%) and right-sided aortic arch with aberrant left subclavian artery may
rarely cause esophageal symptoms (dysphagia lusoria) and are often
demonstrated as chance findings.
The aberrant right subclavian artery arises as the last branch of aortic arch; its
first part is persistent part of distal right arch, which may be larger than its
expected diameter, known as kommerell's diverticulum. This aberrant vessel
passes obliquely upwards and to right, posterior to esophagus, causing posterior
indentation on it. If the vessel is dilated at this point it makes wide indentation.
There is no vascular ring with this anomaly and it is almost always
asymptomatic.
Aberrant left subclavian artery from right aortic arch is the last branch and
passes upwards and to left behind esophagus. A vascular ring is usually formed
by arterial depth joins the left pulmonary artery to the left subclavian artery,
which occasionally may cause significant esophageal compression.
Treatment is indicated for the relief of symptoms and is usually directed at
dividing the nondominant avascular component of the rings.

(Q.135) Upside down stomach on Ba meal study is seen in:


Organoaxial gastric volvulus
(a)
Large Gastric adenocarcinoma at antral end
(b)
Rolling type of hiatus hernia
(c)
Mesentricoaxial gastric volvulus
(d)
Your Response

:
Correct Answer
:
Exp:

D
Mesentricoaxial gastric volvulus

Upside down stomach on Ba meal study is seen in


Mesentricoaxial gastric volvulus.
(Q.136) Ultrasound is investigation of choice for?
Somatostatinoma
(a)
Urethral stricture
(b)
Blunt abdominal trauma
(c)
Intraductal Pancreatic calculi
(d)
Your Response
:
Correct Answer
C
:
Exp:
Blunt abdominal trauma
Ref. Bailey and Love 24th ed. 114
Focused abdominal sonogram for trauma (FAST)
Three most commonly used diagnostic techniques are diagnostic peritoneal
lavage, USG and CT.
DPL is nonspecific, can cause iatrogenic and injury and may not detect
retroperitoneal injuries, while CT is expensive and time consuming which may
also need IV contrast injection and need transporting the patient but, USG
overcomes most of these disadvantages.
Focused assessment with sonography for trauma (FAST) examination is a
bedside screening tool to identify free intraperitoneal, pleural and pericardial
fluid. It is limited ultrasound examination with 6 views.
The average time to perform a complete FAST examination is 24 minutes.

(Q.137) Non contrast spiral CT is investigation of choice for?


Acute ureteric colic
(a)
Acute pulmonary embolism
(b)
Acute mesenteric ischemia
(c)
Acute prolapsed intervertebral disc
(d)
Your Response
:
Correct Answer
A
:
Acute ureteric colic
Exp:
Ref. Bailey and Love 24th ed. Chapter 2 pg. 35

NCCT (Non-contrast spiral/helical CT) is widely used and

currently study of choice for renal colic.


It is accurate, less time consuming, no contrast need, 98%
sensitivity, 97% specificity, no contrast needed, 98% sensitivity,
97% specificity, 100% positive predictive value and 97% negative
predictive value.
Disadvantage: Ionizing Radiations and renal function cannot be
assessed and indenavir calculi can be missed.
(Q.138) Which of the following can be the principle modality of choice for evaluating lungs of a young
patient with suspected alpha1 antitrypsin deficiency?
High resolution Ultrasound
(a)

(b)
(c)
(d)
Your Response
:
Correct Answer
:
Exp:

Arthrography
HRCT
MRI

C
HRCT
Ref. Graingers Diagnostic Radiology 4th ed., 102
HRCT is best to detect and evaluate suspected cases of bronchiectasis,
emphysema, diffuse lung diseases and ILDs.

Applications of MRI are many, but amongst the commonly


imaged parts are brain, spine and musculoskeletal tissues.
MRI is useful in evaluating olfactory bulbs, ventricles, and other
soft tissue of the brain.
Magnetic resonance imaging (MRI) has significantly advanced the
ability to image musculoskeletal structures.
MRI has the advantages of providing multiplanar images with fine
anatomic detail and contrast resolution.
Other advantages are the lack of ionizing radiation and adverse
effects and the superior ability to visualize bone marrow and soft
tissue periarticular structures, which have led to the increased use
of this modality.
The advantages of MRI are counterbalanced by high cost and long
procedural time, factors that have limited its use in the evaluation
of musculoskeletal disorders.
MRI can image fascia, vessels, nerve, muscle, cartilage, ligaments,
tendons, pannus, synovial effusions, and bone marrow.

Because of its sensitivity to changes in marrow fat, MRI is a


sensitive but nonspecific means of detecting osteonecrosis and
osteomyelitis.
Because of its enhanced soft tissue resolution, MRI is more
sensitive than arthrography or CT in the diagnosis of soft tissue
injuries (e.g., meniscal and rotator cuff tears); intraarticular
derangements; and spinal cord damage, subluxation, or synovitis.
(Q.139) X-ray sign of pneumoperitoneum?
(a)
Steeple sign
(b)
Riglers sign
(c)
Golden S sign
(d)
Bird of prey sign
Your Response
:
Correct Answer
B
:
Exp:
Riglers sign
Ref. Bailey and Love 24th ed. pg. 15
The typical finding in sigmoid volvulus is smooth tapered end of contrast
column: hook-like in appearance - bird of prey sign (where the tapered blind
end is seen as a hook).
Laryngotracheobronchitis (croup) is characterized by the narrowing of the
subglottic airway (Steeple sign)
Riglers sign - bowel wall outlined by intraluminal and free peritoneal gas.
The focal bulge at the apex of the collapsed right upper lobe corresponds to the
centrally located bronchogenic carcinoma causing the lobar collapse. The
combined radiologic appearance on frontal radiograph is known as Golden S
sign.

(Q.140) Face of giant panda sign on MRI brain is seen in:


(a)
Wilsons disease
(b)
Japanese encephalitis
(c)
Rasmussens encephalitis
(d)
Wernickes encephalopathy
Your Response
:
Correct Answer
A
:
Exp:
Wilsons disease
Ref. Medical radiology diagnostic imaging by Baert pg. 186

MR IMAGING SHOWS ABNORMALITIES IS WILSONS


DISEASE
Basal ganglia lesions are most often bilateral and symmetrical.
The putamina shows striking increase in T2 signal intensity. This is present to a
lesser degree in other deep gray matter structures.
Thalamic lesions are often present but typically spare the dorsomedial nuclei.
White matter tracts including the dentatothalamic, corticospinal, and
pontocerebellar tracts are commonly involved. The claustrum may show high
T2 signal intensity. The midbrain is bright on T2 weighted images with relative
sparing of its deep nuclei giving rise to the so-called Panda sign.

(Q.141 IOC for acute appendicitis in children


)
USG
(a)
MRI
(b)
CECT
(c)
(d)
X ray
Your Response
:
Correct Answer
A.
:
Exp USG
:
Ref : Emedicine Medscape Pediatric Appendicitis
Author: Robert K Minkes, MD, PhD; Chief Editor: Carmen Cuffari, MD
THIS IS BEING ASKED REPAETEDLY IN AIIMS

Making a timely diagnosis of appendicitis is a difficult challenge in children with abdominal


pain. Laboratory findings may increase suspicion of appendicitis but are not diagnostic.
The minimum laboratory workup for a patient with possible appendicitis includes a white
blood cell (WBC) count with differential and urinalysis.
Liver function tests and amylase and lipase assessments are helpful when the etiology is
unclear. Baseline blood urea nitrogen and creatinine are needed prior to intravenous contrast
CT scanning.
Other studies, such as interleukin 6 and C-reactive protein (CRP) assays, have been advocated
by some in the diagnosis of appendicitis. However, in multiple clinical series, these studies
have not been shown to be of clear benefit and, for the most part, only add to the cost of the
evaluation.

Abdominal Radiograph :
Abdominal radiographyfindings are normal in many individuals with appendicitis. However,
plain films may be helpful in the setting of severe constipation. A calcified appendiceal
fecalith is present in less than 10% of persons with inflammation, but its presence suggests the
diagnosis.
Radiographic signs suggestive of appendicitis include convex lumbar scoliosis, obliteration of
the right psoas margin, right lower quadrant (RLQ) air-fluid levels, air in the appendix, and
localized ileus. In rare incidents, a perforated appendix may produce pneumoperitoneum.
If no other imaging studies are to be performed, an abdominal series may be helpful

Ultrasonography :

Given the potential risks of radiation from CT scans, graded compression


ultrasonography may be the preferred initial imaging modality in the evaluation of
pediatric acute appendicitis. This technique involves locating the appendix and then
attempting to compress its lumen.
For ultrasonography to be diagnostic of appendicitis, the operator must visualize the appendix.
Even if the appendix is not visualized, however, appendicitis can be excluded more
confidently if ultrasonography shows no secondary signs of appendicitis (eg, hyperechoic
mesenteric fat, fluid collection, localized dilated small bowel loop).[4, 5]
A positive finding is a noncompressible tubular structure 6 mm or wider in the RLQ (see the
images below). This structure is tender during palpation with the ultrasonographic probe.
Additional supportive findings include an appendicolith, fluid in the appendiceal lumen, focal
tenderness over the inflamed appendix (sonographic McBurney point), and a transverse
diameter of 6 mm or larger. In patients with a perforated appendix, ultrasonography may
reveal a periappendiceal phlegmon or abscess formation.
Abdominal ultrasonography has proved to be valuable for diagnosing appendicitis in
children, with most published reports indicating a sensitivity, specificity, and accuracy of
at least 90-95%. Furthermore, some authors have found that ultrasonography is more
sensitive and specific than clinical impression and increases diagnostic accuracy when
used either alone or in conjunction with laboratory results.
The advantages of ultrasonography include its noninvasiveness, lack of radiation, no
contrast medium, and minimal pain. The downside of ultrasonography is that the
examination is operator dependent and may not be available at some institutions.
Factors that add difficulty to the examination include obesity and gaseous distention of
the intestines overlying the appendix. However, results of one study determined that
ultrasonography should continue to be the first diagnostic imaging study in suspected
appendicitis, regardless of the child's body mass index (BMI).
Ultrasonography is also useful in diagnosing alternative pathologies (eg, tubo-ovarian abscess,
ovarian torsion, ovarian cyst, mesenteric adenitis). A prospective study by Lowe et al
comparing noncontrast CT scanning and ultrasonography revealed a sensitivity, specificity,
and accuracy of 97%, 100%, and 98%, respectively, for unenhanced CT scanning, compared
to 100%, 88%, and 91%, respectively, for ultrasonography. Still others have shown that the
perforation rate and negative appendectomy rate can be decreased by using both tests in
tandem
CT scan :
CT scanning is a useful modality for diagnosing appendicitis in children. Although
radiation exposure is a concern, CT scans have been shown to have an accuracy of 97% in
diagnosing appendicitis. Advantages of CT scan include the availability at most institutions,
the ability to evaluate the entire abdomen and locate abscesses and phlegmon, the lack of
dependence on operator skill, and physician familiarity with reading CT scans.
CT findings that are indicative of appendicitis include a thickened appendix, fat streaking
around the appendix, or thickening of the cecal wall (see the images below). CT findings that
suggest perforated appendicitis include periappendiceal or pericecal air, abscess, phlegmon,
and extensive free fluid. Because the disease is due to obstruction of the appendix and the
inflammation occurs distal to the obstruction, extravasation of contrast or extensive free air is
rarely seen. If a patient is found to have free air throughout the abdomen or under the
diaphragm, other diagnoses should be entertained. CT scanning may be helpful in obese
patients or those in whom a localized appendiceal abscess is clinically suspected. In patients
with abscesses, CT scanning may also be helpful in the CT-guided drainage of the abscess.
CT scanning has been found to have sensitivity, specificity, and accuracy similar to those of
ultrasonography. Disadvantages include the aforementioned radiation exposure, the need for
oral and intravenous contrast and its related disadvantages, and the need for the patient to be
still, which is often difficult for small children.
Also Teaching Atlas of Pediatric Imaging by Paul Babyn states

Now regarding MRI Note that various studies and papers in various journals agree with the
following :
In children with suspected acute appendicitis, a radiation-free diagnostic imaging of
ultrasonography selectively followed by MRI is comparable to CT with no difference in time to
treatment, negative appendectomy rate, perforation rate, or performance
So USG and MRI may be combined together. If we have to choose 1 out of these it would still be
USG for the following reasons :
USG
MRI
Availability
Readily available
Not so readily available
Cost
Relatively cheaper
Expensive
Feasibility in a sick child
Excellent Can be done
Difficult to perform may require
bedside, No sedation
sedation, specialized immobilization
required, Can be repeated
strategies.
Technical Expertise
Moderate, Available
Specialized, May not be readily
available.

(Q.142) Radiological sign of spondylolysis?


Napoleons hat sign
(a)
'Scottish dog with collar' sign
(b)
Winking owl sign
(c)
Cap sign
(d)
Your Response
:
Correct Answer
B
:
'Scottish dog with collar' sign
Exp:

Ref. Radiology secrets 2nd ed. Chapter 35

On the oblique view of the lumbar spine, an outline of what looks


like a dog can be seen.
The parts of the dog are as follows:
The transverse process makes up the nose, the pedicle makes up
the eye, the superior articular facet makes up the ear, the pars
interarticularis makes up the neck, the inferior articular facet
makes up the front leg, the lamina makes up the body, the
contralateral superior articular facet makes up the tail, and finally,
the contralateral inferior articular facet makes up the hind leg.
Spondylolysis is a defect or fracture of the pars
interarticularis.
On the "Scotty dog" view, it is seen as a band of radiolucency
through the "neck," like a collar.
L5 is most commonly involved.
When this condition involves bilateral pars interarticularis, it is
commonly associated with a condition called spondylolisthesis,
which is anterior translation of the involved vertebral body with
respect to its inferior adjacent vertebral body.
Winking owl sign=Pedical destruction by metatsasis.
(Q.143) Which of the following is earliest sign of ulcerative colitis on double contrast
barium enema (DCBE) study?
(a)
Fine Mucosal granularity
(b)
Loss of haustrations
(c)
Collar-button ulcers
(d)
Lead pipe colon
Your Response
:
Correct Answer
A
:
Exp:
(Fine mucosal granularity)
Double contrast barium enema is the radiological examination of choice to
show disease extent and severity.
It is most accurate technique for demonstration of overall colonic
morphology and has role in long-term management especially when exact
location and extent of any stricture needs to be defined or diffuse colonic
abnormality is to be depicted. Earliest radiological change in ulcerative colitis
(uc) is blurring of mucosal lining and a fine granularity when mucosa is seen

enface due to edema. Abnormal barium adherence to alter colonic mucus and
flecks of barium adhere to superficial erosion.
Instant enema in UC the large bowel is inflammed and contains no fecal
matter, and hence enema study can be done without bowel preparation.

(Q.144) All of the following form radiolucent renal stones except:


Xanthine
(a)
Cysteine
(b)
Uric acid
(c)
Orotic acid
(d)
Your Response
:
Correct Answer D
:
(orotic acid)
Exp:
-----------------------------------------------------------------------------Composition and calculi incidence
------------------------------------------------------------------------------

a. Calcium oxalate

75%

(Spiculated/lamellated/mulberry stones)
b. Struvite (magnesium ammonium phosphate/

15%

Laminated/ proteus induced/staghorn)


c. Calcium phosphate (laminated)

5%

d. Uric acid

5%

e. Cystine; xanthine; orotic acid; matrix

rare

(mucoprotein or mucopolysacchride)
-----------------------------------------------------------------------------Radiolucent renal calculi are:

a. Uric acid stones

b. Xanthine stones

c. Cystine stones (poorly radioopaque)

(Q.145) The following appearance on barium enema is seen with?

d. Matrix (mucoprotein) stones

(a)
(b)
(c)
(d)
Your Response
:
Correct Answer
:
Exp:

Ulcer
Intussuception
Carcinoma
Diverticulum

B
Intussuception
Intussusception with a classical claw sign on barium enema.

(Q.146) Pile of plates appearance on angiography suggests diagnosis of :


Atherosclerosis
(a)
Fibromuscular Dysplasia
(b)
Takayasus Disease
(c)
Moya Moya disease
(d)
Your Response
:
Correct Answer
B
:
Exp:
Fibromuscular Dysplasia
Fibromuscular dysplasia (medial fibroplasias, medial or
fibromuscular hyperplasia, intimal hyperplasia, perimedial
fibroplasias) is the most common cause of renovascular
hypertension in children and young adults and bilateral in 2/3rd
cases.

Angiography: string of beads/Pile of plates appearance of


FMD is classically seen on angiography.
(Q.147) All the following are diagnostic barium follow through features of ileocecal tuberculosis except:
Pulled up contracted caecum
(a)
Widening of ileocecal angle
(b)
Wind-sock appearance of duodenum
(c)
Strictures involving terminal ileum
(d)
Your Response
:
Correct Answer
C
:
Exp:
Wind-sock appearance of duodenum
Ref. Graingers diagnostic radiology, 4th ed., 1087,1134
Intestinal tb: EVERY LINE IS A POTENTIAL QUESTION
The main types of intestinal tb are ulcerative form (most frequent), hypertrophic
form and mixed form.
Location: ileocaecal region > ascending colon > jejunum > appendix >
duodenum > stomach > sigmoid > rectum > esophagus.
Ileocaecal area:
Stierlin sign rapid emptying of narrowed terminal ileum into shortened rigid
obliterated caecum on barium examination.
Thickened ileocaecal valve
Fleischners sign (inverted umbrella defect) wide gaping patulous ileocaecal
immediately adjacent terminal ileum
Deep fissures + large shallow linear/stellate ulcers with characteristic elevated
margins.
Symmetric annular 'napkin ring' stenoses
Widened ileocolic angle (normal is 90o) it becomes obtuse
Colon:
Segmental colonic involvement
Diffuse ulcerating colitis + pseudopolyps
Amputated/ coned/ contracted caecum
Gastroduodenal:

Simultaneous involvement of pylorus + duodenum 'linitus plastic' appearance.


Linitus plastic may also be a feature of gastric lymphoma and scirrhous
carcinoma, eosinophilic gastritis

(Q.148) Earliest sign of left atrial enlargement?


Elevation of the left main bronchus
(a)
Double cardiac shadow
(b)
Widening of carina
(c)
Cephalization of the blood flow
(d)
Your Response
:
Correct Answer
A
:
(elevation of the left main bronchus)
Exp:
Ref. Grainger radiology 4th ed. Pg. No. 684; fig. 32.19; table 32-2

Radiographic signs of left atrial enlargement


The enlarging left atrium may elevate the left main bronchus, which is initially
a rather subjective observation, and if gross may lead to splaying of the carina.
As the enlarging left atrium bulges to the right, from the mediastinum into the
right lung, it produces a double shadow seen through the right side of the heart,
until it eventually forms part of the right heart border. On the right side, the
borders of right and left atria may be distinguished because the left atrial border
passes medially towards the spine before fading, whereas the right atrial border
is limited below by the entry of the inferior vena cava. Other causes of a double
shadow in this area are uncommon but it may be produced by an enlarged aortic
root (continuous with the ascending aorta) or by a confluence of the pulmonary
veins. The right border of the enlarged left atrium may coincide on the frontal
view with the right border of the right atrium; presumably the pericardium
temporarily restrains further enlargement of the left atrium.
Particularly in rheumatic mitral valve disease, there may be left atrial
enlargement with specific enlargement of the left atrial appendage, first seen as
a straightening of the left heart border below the left main bronchus, then as a
discrete bulge on the left, immediately below the pulmonary bay and left main
bronchus. A similar appearance may be seen in congenital defects of the left
pericardium, but these will lack other radiological evidence of left atrial
enlargement as well as clinical signs. After surgical amputation of the left atrial
appendage there will be a concavity at its site on the left heart border, making
the heart shape resemble that in left ventricular enlargement.
Rarely the enlarged left atrium may displace the descending aorta to the left
(bedford sign). It may also displace the oesophagus to the side of the spine,
usually to the right, but rarely to the left, when dysphagia may result.
When the enlarged left atrium bulges to the left and right of the cardiac
silhouette and approaches to within an inch of the lateral chest wall the term
aneurysmal enlargement is used.

(Q.149) Normal angiography but severe hypokinesia of LV wall is seen in?

(a)
(b)
(c)
(d)
Your Response
:
Correct Answer
:
Exp:

HOCM
Restrictive cardiomyopathy
DCM
Tako Tsubu cardiomyopathy

D
Tako Tsubu cardiomyopathy
Tako-Tsubo (Stress) Cardiomyopathy
Abrupt onset of severe chest discomfort preceded by a very
stressful emotional or physical event.
Women >50 years, ST-segment elevations and/or deep T-wave
inversions in the precordial leads.
Normal angiography.
Severe akinesia of the distal portion of the left ventricle with
reduction of the EF.
Troponin are usually mildly elevated.
Cardiac imaging typically shows "ballooning" of the left ventricle
in end-systole, especially of the LV apex. All of these changes,
which are often quite dramatic, are reversible within 37 days and
do not cause long-term cardiac dysfunction or disability.
The mechanism responsible for Tako-tsubo cardiomyopathy is
that an adrenergic surge that includes circulating
catecholamines, acting on the epicardial coronary vessels and/or
coronary microcirculation, is involved.
Beta blockers are used in therapy.

(Q.150) Wrong match regarding the investigation of choice?


Gadolinium enhanced MRI: Superior sulcus tumor
(a)
HRCT: Cryptogenic fibrosing alveolitis
(b)
CECT: Aberrant Right Subclavian artery
(c)
Transesophageal echocardiography: Perioperative myocardia ischemia
(d)
Your Response
:
Correct Answer
C
:
Exp:
CECT: Aberrant Right Subclavian artery

Ref. Alison grainger diagnostic radiology, 4th ed., vol-i, 278


HRCT is of proven value in the diagnosis of diffuse lung disease (like
interstitial lung diseases), particularly in the early stages when the chest
radiograph is normal and for follow-up.
HRCT clearly depicts distribution and higher definition of appearances of
pulmonary parenchymal disease.
Nowadays HRCT is used for detection of bronchiectasis, and surgery is
undertaken without preoperative bronchography. Severity and extent of
bronchiectasis is demonstrated.
HRCT is excellent of detection and evaluation of emphysema also.
HRCT can identify regions most suitable for biopsy at a time when the chest
radiograph is normal.
For aberrant right subclavian artery and its pressure effects MRI would be the
most preferred tool for diagnosis.

(Q.151) Phantom calyx/ Amputed calyx on IVU is feature of?


Tuberculosis
(a)
Hydronephrosis
(b)
Bilharziasis
(c)
UPJ obstruction
(d)
Your Response
:
Correct Answer A
:
Tuberculosis
Exp:
Ref. Radiology of urinary system by elkin, 2nd ed., 149
Early renal tb: IVU
Advanced renal tb: CT
Radiologic findings in renal tb include:

Urographic demonstration (ivu) of diminished excretion of the contrast


medium, focally or generally.
Poorly defined one or more calyces.
Cavitation communicating with the pc system, demonstrable by urography or
retrograde pyelography.the irregular cavities may be multiple.
Cortical scarring with abnormality of underlying calyces, similar to the changes
of chronic pyelonephritis.
Mass lesions due to tuberculomas.

Dilatation of pc system due to parenchymal destruction or strictures. Ureteral


strictures result in generalized pyelocaliectasis. Infundibular (calyceal neck)
stretching and strictures result in localized hydrocalicosis.
Calcification in the renal parenchyma, usually of a granular, mottled type.
However, linear or ring calcification or homogenous amorphous collection are
seen. The calcification rims outlining the periphery of distorted renal lobes is
unique.
Autonephrectomy/putty kidney.
Ureteric tuberculosis
Produces mucosal and wall ulceration, fibrosis, stricture and calcification
saw-tooth or cork-screw ureter
Beaded ureter, short, straight, rigid and narrow ureter especially terminal
segment
Calcification is, however, rare in ureteral tuberculosis and when occurs involves
lower segment
Symmetrical small spastic, and thickened bladder or small multilobular bladder
(thimble bladder).

(Q.152) A female in mid thirties presented with an episode of TIA; on conventional angiography, it showed
following picture. What is the diagnosis?

(a)
(b)
(c)
(d)
Your Response
:
Correct Answer
:
Exp:

Atherosclerosis
Moya Moya disease
Carotid artery aneurysm
History and image is not sufficient

B
Moya Moya disease
Moyamoya syndrome is a disease in which certain arteries in the brain are constricted.
Blood flow is blocked by the constriction, and also by blood clots (thrombosis).
A collateral circulation develops around the blocked vessels to compensate for the
blockage, but the collateral vessels are small, weak, and prone to hemorrhage, aneurysm
and thrombosis.
On conventional X-ray angiography, these collateral vessels have the appearance of a
"puff of smoke"

(Q.153) Bone tumor that typically affects the epiphysis of a long bone?
Osteosarcoma
(a)
(b)
Ewings sarcoma
Chondroblastoma
(c)
Chondromyxoid fibroma
(d)

Your Response
:
Correct Answer
:
Exp:

C
(chondroblastoma)
Ref. Grainger diagnosis of radiology 4th ed. - 1843

Chondroblastoma
Rare neoplasm
Typically located in epiphysis of long bone
Other epiphyseal origin tumors are:
i. Osteoblastoma
ii. Subchondral cyst
Generally benign
80% occur in 2nd decade
Usually located eccentrically in epiphysis but may straddle the
growth plate
Classical chicken-wire calcification may be seen on hp.
X-ray- spherical and well-defined lytic lesion with fine sclerotic
margin.
Central punctate calcification is seen in only 10% cases.
Most common in femur followed by humerus and tibia.
Chondromyxoid fibroma:
10-30 yrs age
Metaphyseal eccentric
Proximal shaft of tibia most classical site.
It is eccentric without lobulations.
Lesion involving cortex and medulla with sclerotic border may show soap bubble
appearance due to endosteal nodding.

(Q.154) Radioisotope of choice for Triple phase Bone scan?


Tc99m-Sestamibi
(a)

(b)
(c)
(d)
Your Response
:
Correct Answer
:
Exp:

Tc99m-Pertectinate
Tc99m-Methyl diphosphonate
Tc99m-Thallium201 subtraction

C
Tc99m-Methyl diphosphonate
Ref. Maheshwari orthopedics 2nd ed. 225
Tc99m-Sestamibi
Tc99m-Pertectinate
and Thyroid scan.

: Parathyroid scan
: Meckles diverticulum, Salivary gland scintigraphy

Tc99m-Methyl diphosphonate
Tc99m-Thallium201 subtraction
adenomas.

: Bone scan
: Second choice for detecting parathyroid

(Q.155) Expansile lytic lesion at skull base typically involving clivus with specks of calcification is suggestive of:
Osteosarcoma
(a)
Secondaries in bone
(b)
Chordoma
(c)
Fibrous histiocytoma
(d)
Your Response
:
Correct Answer
C
:
Exp:
(Chordoma)
Chordoma
It is a destructive bone tumor believed to arise from ectopic
remnants of notochord/ notochord cell rest.
Its maximum incidence between 50-70 years.
It is locally malignant with strong tendency to recur after excision.
Predilection for sacral (50%) and cranial (40%) regions (the
extreme ends of the axial skeleton);
In sacrum at s4 and s5 and in cranium basisphenoid (clivus and
dorsal aspect of sella)
Above sacrum and below c2, chordomas are rare.
Radiologically appears as a oval or lobulated well-defined purely

lytic mass in midline, which may contain calcification and a soft


tissue component.
Recurrence rate is high but it metastasis occurs late.
Chordoma at the base of skull carry best prognosis
(Q.156) Premature closure of coronal, sagittal, and lambdoid sutures results in:
(a)
Oxycephaly
(b)
Turricephaly
(c)
Plagiocephaly
(d)
Trigonocephaly
Your Response
:
Correct Answer
A
:
Exp:
Oxycephaly
Oxycephaly results due to premature closure of coronal,
sagittal, and lambdoid suture.
The most common type of primary craniosynostosis is simple
sagittal synostosis.
Scaphocephaly = Dolichocephaly (55%): premature closure of
sagittal suture (long skull)
Brachycephaly = Turricephaly (10%): premature closure of
coronal / lambdoid sutures (short tall skull)
Plagiocephaly (7%): unilateral early fusion of coronal +
lambdoidal suture (lopsided skull)
Trigonocephaly: premature closure of metopic suture (forward
pointing skull)
Oxycephaly: premature closure of coronal, sagittal, lambdoid
sutures
Cloverleaf skull = Kleeblattschdel: intrauterine premature closure
of sagittal, coronal, lambdoid sutures;
May be associated with: thanatophoric dysplasia
(Q.157) The salt-and-pepper appearance on t1MRI is hallmark of paragangliomas.
The salt and pepper of which represent ___________ respectively.
(a)
Zallballens and multiple flow voids
(b)
Zallballens and discrete calcific foci
(c)
Subacute hemorrhages and calcific foci

(d)
Subacute hemorrhages and flow voids
Your Response
:
Correct Answer
D
:
Exp:
Subacute hemorrhages and flow voids
The salt-and-pepper appearance on t1MRI is hallmark of
paragangliomas. The salt and pepper of which represent
subacute hemorrhages and flow voids respectively.
(Q.158) The most definitive test for identifying intracranial aneurysms is
(a)
MRI scanning
(b)
CT scanning
(c)
Positron emission tomography
(d)
Cerebral angiography
Your Response
:
Correct Answer
D
:
Exp:
Cerebral angiography
Ref. Osborn neuroradiology, pg. 257260.
Computed tomographic scanning is especially sensitive to
intracerebral hemorrhage, but not to aneurysms unless they are
more than 5 mm across. Even such relatively large aneurysms
May not be revealed by CT scanning unless there is bleeding from
the aneurysm or distortion of adjacent structures by the aneurysm.
Microscopic aneurysms may be localizable on CT only because of
the high signal left near the aneurysm by telltale blood. In most
cases of aneurysmal bleeding, angiography is needed to
characterize and localize the lesion.
The resolution of pet, MRI, spect, and CT of intracranial
aneurysms is too poor to enable surgical correction of the lesion to
proceed without demonstration of the aneurysm on angiography.
(Q.159) MARTELS sign is seen in?
(a)
Gout
(b)
SLE
(c)
Takayasu
(d)
Kawasaki disease
Your Response
:
Correct Answer A

:
Exp:

Gout
Cystic changes, well-defined erosions with sclerotic margins
(often with overhanging bony edges), and soft tissue masses are
characteristic radiographic features of advanced chronic
tophaceous gout. Martel sign sharply marginated bony
erosionsfrom a lons standing soft tissue tophous in patient with
gout.

(Q.160) Corpus callosum involvement on MRI is usually seen in which of the following
conditions?
(a)
SSPE
(b)
Krabbes disease
(c)
Phytanic acid deficiency
(d)
Butterfly glioblastoma
Your Response
:
Correct Answer
D
:
Exp:
butterfly glioblastoma
Corpus callosum lesions are characteristically seen in:
a. Adrenoleukodystrophy
disease

b. Marchiafava-bignami

c. Progressive multifocal leukoencephalopathy


e. Toxoplasmosis
Butterfly glioblastoma

f. CNS lymphoma

d. Dai
g.

(Q.161) Positron emission tomographic (PET) scanning is useful in evaluation of the


following except:
(a)
Mycotic aortic aneurysm
(b)
Solitary pulmonary nodules
(c)
Staging lung cancer
(d)
Lymph node involvement by malignancy.
Your Response
:
Correct Answer
A
:
Exp:
Mycotic aortic aneurysm
FDG-PET scan
It is increasingly being used to identify malignant lesions in the
lung based on their increased uptake and metabolism of glucose.

The technique involves injection of a radiolabeled glucose


analogue, 18f-fluoro-2-deoxyglucose (fdg), which is taken up by
metabolically active malignant cells. However, fdg is trapped
within the cell following phosphorylation, and the unstable
fluorine 18 decays by emission of positrons, which can be
detected by a specialized pet camera or by a gamma camera that
has been adapted for imaging of positron-emitting radionuclides.
This technique has been used in the evaluation of solitary
pulmonary nodules and as an aid to staging lung cancer, through
identification of mediastinal lymph node involvement by
malignancy.
(Q.162) Which of the following tumor shows a radiological sign called pneumosinus
dilatans?
(a)
Meningiomas
(b)
Ependymomas
(c)
Hemangioblastomas
(d)
Medulloblastomas
Your Response
:
Correct Answer
A
:
Exp:
Meningiomas
Ref. Osborn, pg 420.
Radiological signs of meningiomas:
- Dural-based tumor

- Hyperostosis of skull

- Pneumosinus dilatans

- Dural tail sign

- Strong homogenous contrast uptake


common
- Mother-in-law sign

- Calcification

(Q.163) A 40 years old female patient presented with recurrent headaches. MRI
showed an extra-axial, dural based and enhancing lesion. The most likely
diagnosis is:
(a)
Meningioma.
(b)
Glioma.
(c)
Schwannoma.
(d)
Pituitary adenoma.
Your Response
:
Correct Answer
A
:

Exp:

Meningioma.
Ref bailey and love surgery 23rd ed. 560
Meningioma is highly vascular brain tumor, especially seen in
middle aged females and is characteristically seen as an extraaxial, dural based and homogenously enhancing lesion.

(Q.164) Pick the wrong match of the following images and the congenital uterine
malformation that it represents ?

(a)

Uterus Bicornis bicollis

(b)

Uterus Bicornis unicollis

(c)

Subseptate Uterus

(d)

Septate uterus
Your Response
:
Correct Answer
D
:
Exp:
Septate uterus.

The HSG image demonstrated here in D demonstrates a


Unicornuate uterus rather than a septate uterus.
TheMllerian duct anomaly classificationis a seven point system that can be
used to describe a number of embryonicMllerian duct anomalies:

Class I:uterine agenesis/uterine hypoplasia


vaginal (uterus: normal/ variety of abnormal forms)
cervical
fundal
tubal
combined
Class II:unicornuate uterus/unicornis unicollis, ~6-25%
communicating contralateral rudimentary horn contains
endometrium
non-communicating contralateral rudimentary horn contains
endometrium
contralateral horn has no endometrial cavity
no horn
Class III:uterus didelphys, ~5-11%
Class IV:bicornuate uterus: next commonest type, ~10-39%
complete division, all the way down to internal the os
partial division, not extending to the os
Class V:septate uterus: commonest anomaly, ~34-55%
complete division, all the way down to internal the os
incomplete division
Class VI:arcuate uterus, ~7%
Class VII:in utero Diethylstilbestrol (DES) exposure:T shaped
uterus

Imaging features of congenital uterine anomalies :


Bicornuate Bicornuate uterus is divided according to the involvement
of thecervical canal:
Uterus
Bicornuate Bicollis: two cervical canals; central
myometrium extends to external cervical os
Bicornuate Unicollis: one cervical canal; central
myometrium extends to internal cervical os
Radiographic features
General
The preferred methods of imaging uterine anomalies
areultrasound,orMRI. The external uterine contour is concave
or heart-shaped, and the uterine horns are widely divergent. The
fundal cleft is typically more than 1cm deep and the
intercornual distance is widened.
The uterus is seen as comprising of caudally fused symmetric
uterine cavities with some degree of communication between
the two cavities (usually at the uterine isthmus). Although not a
specific finding, the angle between the horns of the bicornuate
uterus is usually more than 1053.

Fluoroscopy: hysterosalpingogram (HSG)


A divided uterus can be seen, but it is difficult to differentiate
between septate and bicornuate anomalies since the uterine
fundal contour is not visible5.

MRI

Unicornuate
uterus

May help confirm anatomy by showing a deep (>1 cm) fundal


cleft in the outer uterine contour and an intercornual distance of
>4 cm. The uterus demonstrates normaluterine zonal anatomy.
Aunicornuate uterusorunicornis unicollisis a type ofMullerian
duct anomaly(class II). This type can account for ~10% (range
6-13%) of uterine anomalies and infertility is seen in ~12.5%
(range 5-20%) of cases.
There is failure of onemllerian ductto elongate while the other
develops normally. The embryologic predominance of the
unicornuate uterus to be on the right has not been explained. It
may or may not have rudimentary horn.

Sub-classification
It can be classified into to the following types according to the

Type a:with rudimentary horn the American Fertility


Society3.
a1:horn contains endometrium
a1a:communicating contralateral rudimentary horn
contains endometrium (10%)

a1b:non-communicating contralateral rudimentary


horn contains endometrium (22%)
a2:contralateral horn has no endometrial cavity
(33%)
type b:no horn (35%)
Radiographic features
Ultrasound
Can be difficult to detect on ultrasound. The uterus may be seen
tapering to one side.

Hysterosalpingogram (HSG)
The endometrial cavity usually assumes a fusiform (banana
type) shape (except for type a where there may a small
cavitatory filling defect), tapering at the apex and draining into
a solitary fallopian tube. The uterus is generally shifted off the
midline.

MRI
unicornuate uterus appears curved and elongated,
with the external uterine contour assuming a banana
shape
uterine volume is reduced, and the configuration of
the uterus is asymmetric
normal myometrial zonal anatomy is maintained
Uterus
Didelphys

Classically shows two widely spaced uterine corpora, each with


a single Fallopian tube. Separate divergent uterine horns with
large fundal cleft (as distinct from aseptate uterus)
Uterus didelphys should be differentiated from abicornuate
uterus(separation of horns only) and aseptate uterus(midline
uterine septum).

Hysterosalpingogram (HSG)
HSG demonstrates two separate endocervical canals that open
into separate fusiform endometrial cavities, with no
communication between the two horns. Each endometrial
cavity ends in a solitary fallopian tube.
If the anomaly is associated with an obstructed longitudinal
vaginal septum, only one cervical os may be depicted, and it
may be cannulated with the endometrial configuration
mimicking a unicornuate uterus.

Pelvic ultrasound
Separate divergent uterine horns are identified with a large
fundal cleft. Endometrial cavities are uniformly separate, with
no evidence of communication. Two separate cervices need to

be documented.

MRI
MR imaging demonstrates two separate uteri with widely
divergent apices, two separate cervices, and usually an upper
vaginal longitudinal septum. In each uterus, normal uterine
zonal anatomy is preserved.

Septate
uterus

General
the external uterine fundal contour may be convex,
flat, or mildly (< 1 cm) concave
acute angle <75 degrees between uterine cavities
endometrial canals are completely separated by
tissue iso-echoic to myometrium with extension into
endocervical canal
Ultrasound
The echogenic endometrial stripe is separated at the fundus by
the intermediate echogenicity septum. The septum extends to
the cervix in a complete septate uterus. The external uterine
contour must demonstrate a convex, flat, or mildly concave
(ideally <1cm) configuration and may best be appreciated on
coronal images of the uterus.

Colour Doppler
May show vascularity in the septum in 70% of cases; and if
present may be associated with a higher rate of obstetric
complications8

Fluoroscopy - Hysterosalpingogram
Accuracy of hysterosalpingogram alone is only 55% for
differentiation of septate uterus from bicornuate uterus. An
angle of less than 75between the uterine horns is suggestive of
a septate uterus, and an angle of more than 105 is more
consistent with bicornuate uteri. Unfortunately, the majority of
angles of divergence between the horns fall between these
ranges, and considerable overlap between the two anomalies is
noted.

Pelvic MRI

Arcuate
uterus

MRI is considered the current imaging modality of choice.


On MR images, the septate uterus is generally normal in size
and each endometrial cavity appears smaller than the
configuration of a normal cavity.
The septum may be composed of fibrous tissue (low T2 signal
intensity), myometrium (intermediate signal), or both2.
General features include:

normal fundal contour


no division ofuterine horns

smooth indentation of fundalendometrial canal: the


depth of indentation is usually considered to be <1
cm
increased transverse diameter ofuterine cavity
Pelvic ultrasound
A normal external uterine contour is noted, with a broad
smooth indentation on the fundal segment of the endometrium.
No division of the uterine horns.

Hysterosalpingogram
Opacification of the endometrial cavity demonstrates a single
uterine canal with a broad saddle-shaped indentation of the
uterine fundus.

MRI

T shaped
uterus

A normal external uterine contour is maintained. The


myometrial fundal indentation is smooth and broad, and the
signal intensity of this region is isointense to normal
myometrium.
T-shaped uterusrefers to a specific radiographic appearance of
the endometrial cavity.
It is the most commonly associated abnormality from in utero
diethylstilbestrol (DES) exposure, seen in 31% of exposed
women. It is classified as a class VIIMllerian duct anomaly.
DES is a synthetic oestrogen that was introduced in 1948 and
was prescribed for women experiencing recurrent spontaneous
abortions, premature deliveries, and other pregnancy
complications. By increasing the synthesis of placental steroidal
hormones, DES was thought to decrease the frequency of
pregnancy loss.

Hysterosalpingogram (HSG)
Typically shows a narrowed irregular endocervical canal. The
opacified endometrial cavity appears small, with a shortened
upper uterine segment, resulting in the characteristic T
configuration.

Complications
In utero exposure to DES was shown to be associated withclear
cell carcinoma of the vagina/cervix. The use of the drug was
therefore abruptly discontinued in 1971 (incidence 0.14-1.4 per
1000 women exposed).

(Q.165) Lactating women with painful breast, 1st investigation to be done should be?
(a)
USG.
(b)
Mammography
(c)
CT
(d)
MRI
Your Response
:

Correct Answer
A
:
USG
Exp:
A mammogram is not the best diagnostic tool for a lactating woman for the
same reasons that it is not generally recommended for most women under age
35 years. BUT IT IS THE FIRST INVESTIGATION FOR A MASS IN
LACTATING WOMAN.
Young women tend to have very dense breasts, making it difficult to find
radiographic lesions. The lactating breast shows an even greater increase in
parenchymal density, with more nodular and rope-like characteristics,
corresponding to ductal distention with milk.
Here is an alternative approach to consider for lactating women with suspected
lesions[1]:
Begin with an ultrasound if there is a palpable abnormality. Ultrasound
can clearly demonstrate a solid vs cystic mass or galactocele. This would
initially eliminate the need for a mammogram.
Fine-needle aspiration can be offered if a mass is determined to be a cyst or
galactocele, usually with no further intervention necessary.
If the palpable mass appears solid on ultrasound, a biopsy is indicated.
Stereotactic guidance for fine-needle aspiration or large-core breast biopsy can
be used successfully in the lactating woman. To minimize creation of a milk
fistula, the affected breast should be pumped just prior to the procedure.
Biopsy is also indicated if the mass is palpable but the ultrasound is negative.
If the abnormality is found to be malignant, then a bilateral mammogram is
done, the purpose of which is to explore whether there are other suspicious
lesions.
Fortunately, breast cancers are not common in lactating women. Lactating
adenomas and fibroadenomas are the most common solid palpable masses and
these are always benign.
However, when malignancies are found, they are often in an advanced stage
with a poor prognosis. This has less to do with the physiology of these lesions
and more to do with the delay in identifying them.
Although routine mammography screening is not indicated, a baseline clinical
breast examination is an important part of initial prenatal care.
Changes or abnormalities of any kind should be investigated with the same
level of concern regardless of whether a woman is breast-feeding.
The National Comprehensive Cancer Network's guidelines of first using breast
ultrasound followed by mammography when the sonographic findings are
inconclusive or suspicious in the evaluation of nonpregnant symptomatic
women younger than 30 years appears to be an appropriate approach to the
evaluation of symptomatic pregnant or lactating women.

CT is very rarely used in breast imaging. MRI is used as a


problem solving modality when USG / Mammography fails to
give a definitive diagnosis. Hence if the 1st investigation in a
lactating female is the question asked, USG should be the best
option.
(Q.166) Investigation of choice in stress fracture
(a)
MRI
(b)
CT
(c)
Bone scan
(d)
X ray
Your Response
:

Correct Answer
A
:
Exp:
MRI
Now conventional wisdom and also all the elderly examiners will
spontaneously opt for Bone scan as the answer. However a careful
review of literature preferentially indicates MRI to be a much
better modality highly sensitive as well as specific for
diagnosis of stress fractures. Go with MRI.
Most reliable reference here would be:
Title: Rockwood & Green's Fractures in Adults, 6th Edition, Chap
21
The diagnosis of a stress fracture often does not require imaging
studies, especially when a careful history and a classic physical
examination combine to make the diagnosis with certainty;
however, several radiographic modalities are at the disposal of the
clinician for definitive documentation and differential evaluations.
Plain x-rays, bone scintigraphy, computed tomography (CT), and
magnetic resonance imaging (MRI) scans are now the routine
examinations that are used to evaluate and diagnose stress
fractures.
X-rays obtained very early in the stress fracture
Xrays
process typically are not effective in demonstrating
an abnormality. Findings rarely appear before 2 to 3
weeks from the onset of symptoms. New periosteal
bone formation, the classic radiographic marker of a
healing response, often does not appear until 3
months from symptom onset. In some patients,
radiographic changes never appear for a number of
stress fractures . When changes are evident, a
number of findings confirm the presence of a stress
fracture, rendering this modality poorly sensitive but
highly specific. Only 20% of bone scan foci positive
for stress fractures correlate with positive plain film
findings . The false-negative rate for x-rays
approaches 100% for early grade I bone scan
positive lesions, but drops to 24% for grade III
lesions, demonstrating that stress fractures have more
ability to remodel and respond to the altered stresses
later in their course, and the later response is often
apparent on plain x-rays.
Findings associated with fractures may include
periosteal bone formation, horizontal or oblique
linear patterns of sclerosis, endosteal callus, and a
frank fracture line. The initial radiographic sign of a

progressing stress fracture is the gray cortex, which


corresponds to a low-density cortical area affected by
increased osteoclastic bone resorption activity. As
the process evolves in long bones, the stress fracture
undergoes marginal resorption and may yield an
ovoid lucency within a thickened area of cortical
hyperostosis. A late-stage stress fracture in cortical
bone appears as a radiolucent line with extension
partially or completely across the cortex. Similarstage stress fractures in cancellous bone demonstrate
a fracture lucency oriented perpendicular to the
trabeculae. Healing is noted by focal sclerosis in
areas of cancellous bone, whereas diaphyseal healing
involves both periosteal and endosteal cortical
thickening . Plain films are most likely to present
positive findings in the fibula and metatarsals. Films
should be centered over the painful, suspected bone
and typically are not helpful in discerning stress
injuries to the pars interarticularis. Some
investigators contend that plain films are unlikely to
yield positive results when investigating possible
tibial stress fractures, and others state the femur,
pars, and tarsal bones are least likely to yield
remarkable findings on initial plain film
investigation.
Scintigraphy When x-ray findings are conclusive, additional
/ Bone scan studies are not required. If multiple sites of stress
fracture are possible based on history and physical
examination, or if plain films do not support the
presumptive diagnosis of stress fracture, three-phase
bone scintigraphy has been the study of choice. Bone
scan has long been considered the most sensitive
test for stress fracture, with sensitivity
approaching 100%, but the sensitivity is not
coupled with high specificity, so clinical features
must be correlated.
Isotopes are atoms with identical atomic numbers but
different atomic weights, and a nuclide is the nucleus
of a given isotope. Nuclides or isotopes with
differences in numbers of protons and neutrons are
unstable and give off particles or electromagnetic
radiation in their transition to stability known as
radioactive decay. These materials are synonymously
called radionuclides or radioisotopes. When used for
diagnostic purposes, the materials are termed
radiopharmaceuticals and radiotracers.
Technetium-99m methylene diphosphonate (99mTc

MDP) is the usual radioisotope utilized for bone


scans. Gamma radiation is emitted, and the whole
body dose for a bone scan is about 0.13 to 0.19 rad.
The bladder dose, where the radioactivity is
concentrated in the urine, is 2.62 to 3.90 rads, hence
the need for frequent voiding during and after the
scanning period.
A grading system, based on the scintigraphic
appearance, allows classification into milder or more
severe stress fractures, recognizing that these stress
injuries occur along a continuum of bony
involvement (Table 21-1) (78). The minimally
symptomatic grade 1 or grade 2 stress fractures
typically resolve more quickly and completely. The
grading system can assist in prescribing the requisite
rest and rehabilitation intervals (27).

MRI

Single-photon emission CT (SPECT scanning) is


used when lumbosacral stress fractures are
suspected. The camera rotates about the patient,
generating 3-D images of radioisotope uptake. This
modality is particularly useful for investigating
suspected pars interarticularis and sacral stress
fractures
MRI scans provide identical sensitivity and
superior specificity compared to bone
scintigraphy for evaluating stress fractures.
The improved specificity derives from the
comprehensive anatomic visualization provided from
this modality, allowing for precise localization of the
injury and differentiation from other possible
conditions. The bony tissue, with comparatively few
mobile protons, is not represented in significant
detail. Instead, the MRI scan accentuates reactive
edema in the soft tissues and marrow surrounding a
stress injury. This soft tissue response is seen best in
edema-sensitive sequences such as fat-suppressed
T2-weighted and short tau inversion recovery (STIR)
scans. Areas of edema appear as high-signal intensity
sites on these sequences.
Although plain x-rays and bone scintigraphy
accurately diagnose stress fractures in most
patients, MRI scans are increasingly advocated as
the study of choice because they do not involve
exposure to ionizing radiation and require much
shorter imaging times than bone scintigraphy.

CT

The specificity of MRI scans significantly exceeds


that of scintigraphy; for this reason,
straightforward cases can be investigated reliably
without MRI, but more difficult diagnostic
dilemmas or problematic cases may well warrant
MRI scanning.
CT scans provide excellent bony anatomic detail.
The delineation of fracture line orientation is assisted
with 3-D CT information, which may improve
treatment decisions for certain bones such as the
tarsal navicular (87,88). Longitudinal fracture lines
in diaphyseal locations can also be elucidated. Pars
and sacral stress fractures are also well characterized
with CT scans
Bone Scintigraphy and MRI Grading Scale

Grade
Bone Scan
1
Small, ill-defined
cortical area of
mildly increased
activity
2

MRI
Periosteal edema: mild to moderate
on fat-suppressed T2 or STIR
images; marrow is normal on T1
and fat-suppressed T2 or STIR
images
Better-defined
Periosteal edema: moderate to
cortical area of
severe on fat-suppressed T2 or
moderately increased STIR images; marrow edema on
activity
fat-suppressed T2 or STIR images
Wide to fusiform,
Periosteal edema: moderate to
cortical-medullary severe on fat-suppressed T2 or
area of highly
STIR images; marrow edema on
increased activity
T1 and fat-suppressed T2 or STIR
images
Transcortical area of Periosteal edema: moderate to
intensely increased severe on fat-suppressed T2 or
activity
STIR images; marrow edema on
T1 and fat-suppressed T2 or STIR
images; fracture line clearly visible

So from the above discussion it may be derived that


For a case of Stress fracture :
1st modality done
IOC - Overall
IOC Clinically strong
suspicion
IOC Complicated history /
Diagnostic dilemma
(Q.167) The unit of absorbed dose equivalent is ?

Xray
MRI
Bone scintigraphy
MRI

RAD
(a)
REM
(b)
Gray
(c)
Millicurie
(d)
Your Response
:
Correct Answer
B
:
(Rem)
Exp:
Ref. Park PSM, 17th ed. 363, 522
Radiation units
Quantity
Activity
Exposure
Absorbed dose
Dose equivalent

Old unit
Curie
Roentgen
Rad
Rem

New si unit
Becquerel
Coulomb/kg
Gray
Sievert

(Q.168) Following are indications for post-operative radiotherapy in a case of carcinoma endometrium, except:
Myometrial invasion of more than half thickness
(a)
Positive lymph nodes
(b)
Endocervical involvement
(c)
Enlarged uterine cavity
(d)
Your Response
:
Correct Answer
D
:
Enlarged uterine cavity
Exp:
Standard treatment options for stage i endometrial cancer:
If the tumor is well or moderately differentiated, involves the upper 66% of the corpus,
has negative peritoneal cytology, is without vascular space invasion, and has <50%
myometrial invasion, a total abdominal hysterectomy and bilateral salpingooophorectomy should be done. Selected pelvic lymph nodes may be removed. If they are
negative, no postoperative treatment is indicated. Postoperative treatment with a vaginal
cylinder is advocated by some clinicians.
For all other cases and cell types, a periaortic and selective pelvic node sampling should
be combined with the total abdominal hysterectomy and bilateral salpingooophorectomy, if there are no medical or technical contraindications. One study found
that node dissection per se did not significantly add to the overall morbidity from
hysterectomy. While the radiation therapy will reduce the incidence of local and regional
recurrence, improved survival has not been proven and toxic effects are worse. Results
of two randomized trials on the use of adjuvant radiation therapy in patients with stage i
disease did not show improved survival but did show reduced locoregional recurrence
with an increase in side effects.
If the pelvic nodes are positive and the periaortic nodes are negative, total pelvic
radiation therapy, including the common iliac nodes, should be given. The incidence of
bowel complications is approximately 4%, and it can be even higher if the radiation
therapy is given after pelvic lymphadenectomy. If the surgery is done using a
retroperitoneal approach, the toxic effects are lessened. If the periaortic nodes are
positive, the patient is a candidate for clinical trials that could include radiation therapy

and/or chemotherapy. Patients who have medical contraindications to surgery should be


treated with radiation therapy alone, but inferior cure rates below those attained with
surgery may occur.

(Q.169) Tc-99 is derived from:


(a)
(b)
(c)
(d)
Your Response
:
Correct Answer
B
:
(mo-99)
Exp:

Str-99
Mo-99
Str-90
Mo-90

Ref. Rrm dahnert 5th ed. 1067


Production of radionuclides
Mo-99/tc-99m generator mo-99 (67 hours) tc-99m (6 hours) tc-99 (2.1x105 years)
ru-99 (stable)
Kr-81m generator

(Q.170) Multiple gated radionuclide cardiac (MGA) scan is used for all except ?
Testing drug toxicity on myocardium
(a)
Myocardial perfusion assessment
(b)
Testing ventricular function
(c)
Detecting myocardial aneurysm
(d)
Your Response
:
Correct Answer
A
:
(testing drug toxicity on myocardium)
Exp:
(Ref. Radiology review manual, 5th ed., 1100)
MUGA scan (multiple gated acquisition scan)

In multiple gated acquisition (MUGA) for cardiac imaging, gated


equilibrium images depict average cardiac contraction by summation over
several minutes.
Recording of ejection fraction of left ventricle before and after exercise,
regional wall motion of ventricular chambers and regurgitant index is done.
Recording of:
Ejection fraction (EF) of left ventricle before + after exercise (>6 million counts, 32
frames)
Regional wall motion of ventricular chambers (>4.5 million counts, 24 frames)

At rest: myocardial infarction, aneurysm, contusion


During exercise: ischemic dyskinesia (detectable in 63%)
Regurgitant index
Projection:
Best septal view (usually LAO 45) for EF; often requires some cephalad tilting of
detector head
Two additional views for evaluation of wall motion (usually anterior + left lateral views)

Advantages of muga scan are:


- Higher information density than 1st pass method
pharmacological effect possible

- Assessment of

- "Bad beat" rejection possible


Disadvantages of muga scan are:
- Significant background activity
chambers

- Inability to monitor individual

- Plane of av valve difficult to identify

(Q.171) LAW OF RADIOBIOLOGY involves 4 Rs, which are following except:


Repair
(a)
Reoxygenation
(b)
Repopulation
(c)
Reperfusion
(d)
Your Response
:
Correct Answer
D
:
Reperfusion
Exp:
Ref. Harrisons principles of medicine 15th ed. 2562
LAW OF RADIOBIOLOGY involves following 4 Rs, which are:
a. Repair

b. Reoxygenation

(Q.172) Earliest feature of radiation toxicity is:


Erythema
(a)
Desquamation
(b)
Edematous rough skin
(c)
Hyperpigmentation
(d)
Your Response
:
Correct Answer A

c. Repopulation

d. Redistribution

:
Exp:

(erythema)
Erythema blackening desquamation dermatitis ulcers
Skin reaction can be seen with two weeks of fractionated radiotherapy (rt).
Erythema is the earliest observed feature followed by desquamation (dry/moist).
Chronic reaction can be seen starting at 6-12 months after irradiation.
Atrophic, easily damageable epidermis, hyperpigmentation, thin skin with hair loss also
occurs.
Rarely malignant change can occur.

(Q.173) Acute myocardial infarct scintigraphy is done with:


Thallium
(a)
Gallium
(b)
Neodymium
(c)
Tc99m stannous pyrophosphate
(d)
Your Response
:
Correct Answer
D
:
Exp:
(tc99m stannous pyrophosphate)
Myocardial infraction imaging with tc-pyrophosphate has a
sensitivity of at least 85% for detection of MI when performed
1-3 days after event.
Specificity is lower.
In clinical practice, this technique is not used commonly but can
be useful in doubt.
Antimyosin antibodies labeled with indium has been used and has
high sensitive and specificity.
Tc labeled glutaric is introduced recently which overcome
limitation of 99m tc pyrophosphate and indium-11 labeled
antimyosin.
(Q.174) Local pain can be treated with the following except
(a)
Portable MRI
(b)
Diathermy
(c)
Ultrasound therapy
(d)
Transcutaneous electrical stimulation
Your Response
:

Correct Answer
A
:
Exp:
Portable MRI
Finally, since older patients generally tolerate aspirin and other
nsaids less well than do younger patients, localized pain should be
treated when possible with local measures such as injection,
physical therapy, heat, ultrasound, or transcutaneous electrical
stimulation.
(Q.175) Gastroparesis is most commonly diagnosed with
(a)
MRI abdomen
(b)
Multi detector CT
(c)
Scintigraphic scan
(d)
High frequency ultrasound
Your Response
:
Correct Answer
C
:
Exp:
scintigraphic scan
Anatomic studies may be indicated if initial testing is
nondiagnostic. Upper endoscopy detects ulcer or malignancy, and
small-bowel barium radiography diagnoses partial small-bowel
obstruction. Colonoscopy or contrast barium enema can detect
colonic obstruction. Abdominal ultrasound or computed
tomography (CT) defines intraperitoneal inflammatory processes,
while CT or magnetic resonance imaging (MRI) of the head can
delineate intracranial sources of nausea and vomiting.
Mesenteric angiography or MRI is useful when ischemia is
considered.
Gastrointestinal motility testing may detect a motor disorder that
contributes to symptoms when anatomic abnormalities are absent.
Gastroparesis is most commonly diagnosed with gastric
scintigraphy, by which emptying of a radiolabeled meal is
measured. Electrogastrography, a noninvasive method to test
gastric slow-wave activity using cutaneous electrodes placed over
the stomach, has been proposed as an alternative means of
diagnosing gastroparesis. The diagnosis of intestinal
pseudoobstruction is suggested by abnormal barium transit on
small-bowel contrast radiography. Small-intestinal manometry
may provide confirmation of the diagnosis and further characterize
the motor abnormality as neuropathic or myopathic based on
contractile patterns. Such investigation can obviate the need for
open intestinal biopsy to evaluate for smooth-muscle or neuronal
degeneration.

(Q.176) Prophylactic craniospinal irradiation is used as treatment modality for


following except:
Small cell carcinoma lung
(a)
Acute leukemic leukemia
(b)
Hodgkin's disease
(c)
Non Hodgkin's lymphoma
(d)
Your Response
:
Correct Answer C
:
(Hodgkin's disease)
Exp:
Ref. Harrison's medicine, 16th ed., 2449
Lung cancer, breast cancer, leukemias, NHL, gastrointestinal malignancies and melanoma are
common tumors that metastasize to brain by contrast, prostate and ovarian cancer and
Hodgkin's disease (HD) rarely metastasize to brain.

Radiation Is The Primary Treatment For Brain Metastases. Thus,


Prophylactic Craniospinal Irradiation Is Useful In Lung Cancer,
All and NHL But Not In HD.
(Q.177) Which of the following is used in treatment of differentiated thyroid cancer?
(a)
131i
(b)
99m tc
(c)
32p
(d)
131i-mibg
Your Response
:
Correct Answer
A
:
Exp:
(131i)
Ref. Sutton radiology 7th ed. 1504
Iodine has 25 isotopes, only of which (127i) is stable, the rest are
radioactive.
131i is favoured for therapeutic purposes because of its emission and a half-life of 8 days.
123i has the most favourable properties for imaging, including a
half life of 13 hours, no emission and a principal gamma
emission at 159kev.it is preferred for imaging retrosternal or
ectopic thyroid tissue, in neonatal or childhood hypothyroidism,
and in the follow up of patients who have had surgery for thyroid
malignancy.
Pertechnetate is widely used for routine thyroid imaging as a
substitute for iodine because 99mtc is readily available and

inexpensive, and radiation dose delivered is relatively low.


(Q.178) Phosphorous-32 emits:
Beta Particles
(a)
Alpha Particles
(b)
Neutrons
(c)
X-Rays
(d)
Your Response
:
Correct Answer
A
:
Exp:
(Beta Particles)
Ph-32 Is Usually Administered Intravenously And Used In The
Treatment Of Polycythemia Rubra Vera (Prv) And Related Blood
Disorders. Its Half Life Is 14 Days. It Is Pure Beta Emitter.
Predominat beta emitters include:
- Phosphorous-32

- Strontium-89

- Yttrium

- I131

(Q.179) Which of the following radioactive isotope is not used for brachytherapy?
(a)
Iodine 125
(b)
Iodine 131
(c)
Cobalt 60
(d)
Iridium 192
Your Response
:
Correct Answer
C
:
Exp:
Cobalt 60.
Brachytherapy is a form of treatment that uses direct placement of
radioactive sources or materials within tumors (interstitial
brachytherapy) or within body or surgical cavities (intracavitary
brachytherapy), either permanently (allowing for full decay of
short-lived radioactive materials) or temporarily (either in one
extended application or over several shorter-term
Pharmaceutical
Dose keV
T1/2 T1/2
Organ
phys bio
Brain
Tc-99m pertechnetate
30 mCi 140
6h
Tc-99m DTPA
10 mCi 140
6h
Tc-99m glucoheptonate 10 mCi 140
6h
Tc-99m Ceretec
20 mCi 140
6h
I-123 Spectamine
36 mCi 159
13.6 h
CSF
In-111 DTPA
500
173, 2.8 d
Ci 247

Tc-99m DTPA
Cardiac Tl-201

1 mCi 140
6h
12 mCi 72,
73 h
135,
167
Tc-99m pyrophosphate 15 mCi 140
6h
Tc-99m pertechnetate
15-25 140
6h
mCi
Tc-99mlabeled RBCs 10-20 140
6h
mCi
Tc-99m sestamibi
25 mCi 140
6h
Tc-99m teboroxime
30 mCi 140
6h
Liver
Tc-99m sulfur colloid
3-5
140
6h
mCi
Tc-99m DISIDA
4-5
140
6h
mCi
Lung
Xe-127
5-10
172, 36.4 d 13 s
mCi
203,
375
Xe-133
10-20 81, 161 5.3 d 20 s
mCi
Kr-81m
20 mCi 176, 13 s
188,
190
Tc-99m MAA aerosol 3 mCi 140
6h
8h
Kidney Tc-99m DTPA
15-20 140
6h
mCi
Tc-99m DMSA
2-5
140
6h
mCi
Tc-99m glucoheptonate 15-20 140
6h
mCi
Tc-99m
10 mCi 140
6h
mercaptoacetyltriglycine
I-131 Hippuran
250
365* 8 d
18 m
Ci
I-123 Hippuran
1 mCi 159
13.2
Thyroid Tc-99m pertechnetate
5-10
140
6h
mCi
I-123
50-200 159
13.2 h
Ci
I-125
30-100 27, 35 60 d
Ci
I-131
30-100 365* 8 d
Ci
Testes Tc-99m pertechnetate
10 mCi 140
6h
Gastric Tc-99m pertechnetate
50
140
6h
mucosa
Ci /
kg

Gallium Ga-67 citrate

WBC

In-111 oxine

3-5
mCi

550
Ci
Tc-99m Ceretec
10-20
mCi
mnemonic: * = as many days as in a year

93,
184,
296,
388
173,
247
140

3.3 d

2.8 d
6h

Also iridium 192 is used in gynaecologic intracavitatory


treatments.
Cobalt 60 is used in the conventional teleradiotherapy and not in
brachytherapy.
(Q.180) Total body Electron beam therapy may be used in treatment of?
(a)
Mycosis fungoides
(b)
Brain tumors
(c)
Carcinoma breast
(d)
All of the above
Your Response
:
Correct Answer
A
:
Exp:
mycosis fungoides
Ref. Harrison's principles of internal medicine 16th ed. 435
Particulate forms of radiation are also used in certain
circumstances.
Electron beams have a very low tissue penetrance and are
used to treat skin conditions such as mycosis fungoides.
Neutron beams may be somewhat more effective than x-rays in
treating salivary gland tumors.
(Q.181) Isotopes used for teleradiotherapy are:
Radon
(a)
Cobalt-60
(b)
Iridium
(c)
Iodine-125
(d)
Your Response
:
Correct Answer
B
:
Cobal-60
Exp:

Following are the three basic methods of treating with radiotherapy:


a. Teletherapy

b. Brachytherapy

c. Systemic radiotherapy

Teletherapy or external beam radiation therapy

X-ray beams
Conventional
i. Superficial: 40 to 120 kv

ii. Orthovoltage: 250 to 400 kv

Megavoltage: > 1 mev


Supervoltage: 2, 4, 6, 12 and 35 mev
Gamma ray beams:
Particle beams :

i. Cobalt 60 beam

i. Electrons

ii. Protons

ii. Cesium 137 beam


iii. Neutrons

iv. Deuterons

(Q.182) Which of the following is not radiotherapy equipment?


Betatron
(a)
Tele curie-cobalt unit
(b)
Linear accelerator
(c)
Thimble chamber
(d)
Your Response
:
Correct Answer
D
:
(thimble chamber)
Exp:
Teletherapy can be carried out using a gamma beam from a radioactive cobalt-60 source
(cobalt unit); though in developed countries the much more versatile linear accelerators
are used as high energy x-ray source. Betatron and cyclotron are also used as the
radiotherapeutic equipments.

(Q.183) Unilateral lung hyperlucency on CXR:


(a)
Chylothorax
(b)
Massive consolidation
(c)
Absent lung
(d)
Absent pectoralis major
Your Response
:
Correct Answer
D
:
Exp:
Absent pectoralis major

(Q.184) Most sensitive investigation for air embolism:


(a)
End tidal co2
(b)
End tidal n2o
(c)
Doppler ultrasound
(d)
CVP
Your Response
:
Correct Answer
A
:
(end tidal co2)
Exp:
Ref. Morgans anesthesia 3rd ed. 575
Air embolism (mill-wheel murmur) can be detected by measurement of end
tidal co2 and pulmonary artery pressure before any clinical signs appear.

(Q.185) Gold standard study for diagnosis of pulmonary embolic episode is:
(a)
Ventilation perfusion scan
(b)
Pulmonary angiography
(c)
Chest x-ray
(d)
Multislice CT
Your Response
:
Correct Answer
B
:
(Pulmonary angiography)
Exp:
Although pulmonary angiography is supposed to be the definitive and gold standard
means of diagnosing PULMONARY EMBOLISM with the decreasing availability of
pulmonary arteriography and increasing use of other non-invasive tests like spiral CT,
the exact role of pulmonary arteriography is now less clear and there are only few
indications for it, especially in absence of spiral ct or MRI.

(Q.186) The typical movement of mitral valve calcification is:


Upwards and downwards
(a)
Counter clock wise
(b)

(c)
(d)
Your Response
:
Correct Answer
:
Exp:

Side to side
Circular

C
(side to side)
The aortic valve is oriented near the horizontal plane and usually overlies the spine,
while the mitral valve is more vertically positioned and lies to the left of the spine.
However, because the valves have a common insertion, it may be difficult to distinguish
calcification of one valve from the other in the frontal view. This can be resolved with
fluoroscopy in the same projection, because the motion of the aortic valve during the
cardiac cycle is near to the vertical while the mitral valve moves more from side to side.

(Q.187) Which phase of cell cycle is most sensitive to radiation?


M
(a)
G1
(b)
G2
(c)
S
(d)
Your Response
:
Correct Answer
A
:
M phase
Exp:
Ref. Principles of radiation therapy by deeley, 27
M phase followed by g2-m interphase is when the cell is radiosensitive. (g2m
interphase > m > g2)
The cell is vulnerable to radiation in the stage of mitosis (m), less so during synthesis (s)
and relatively insensitive during resting periods.
Radiosensitivity of a cell also depends on its histological type and oxygenation of the
tissues.

(Q.188) Single most important advantage of MRI?


Excellent inherent contrast resolution
(a)
Short acquisition time
(b)
Excellent delineation of fine details of the bony cortex
(c)
Magnetic susceptibility effect
(d)
Your Response
:
Correct Answer
A
:
Excellent inherent contrast resolution
Exp:
The important advantages of MRI
a. Multiplanar capability

b. High contrast resolution

c. No use of ionizing radiations

d. Vessels can be seen without use of contrast

e. Functional imaging possible

f. No beam-hardening artefacts

(Q.189) MRI is contraindicated in patients with:


Metallic foreign body in eye
(a)
Sensitivity to dye
(b)
Intracranial hemorrhage
(c)
Agaro phobia
(d)
Your Response
:
Correct Answer
A
:
Metallic foreign body in eye
Exp:
Applications of MRI are many, but amongst the commonly imaged parts are
brain, spine and musculoskeletal tissues.
Contraindications to MRI:
a. Cardiac pacemakers
valves

b. Cochlear implants

d. Intraoccular metallic foreign body


Claustrophobia

c. Prosthetic heart

e. Aneurysmal clips

f.

(Q.190) Which is Not radiological sign of pulmonary embolism?


Palla's sign
(a)
(b)
Hampton's hump
McConnells sign
(c)
Condana effect
(d)
Your Response
:
Correct Answer
D
:
Condana effect
Exp:
A normal or near-normal chest x-ray in a dyspneic patient suggests pulmonary
embolism. Well-established abnormalities include focal oligemia (westermark's sign), a
peripheral wedged-shaped density above the diaphragm (hampton's hump), or an
enlarged right descending pulmonary artery (palla's sign).

(Q.191) The intracranial material appearing most dense on computed tomography (CT) of the head is which of
the following?

(a)
(b)
(c)
(d)
Your Response
:
Correct Answer

Acute hematoma
White matter
Gray matter
CSF

:
Exp:

Acute hematoma
Ref. Osborn neuroradiology, pg 158159.
Computed tomographic scanning measures the density of intracranial as well as
extracranial structures. Bone appears much denser than blood, but blood is obvious on
the unenhanced (precontrast) CT scan precisely because it is much denser than white
matter, gray matter, and CSF. The resolution of the CT scan is generally not sufficient to
differentiate the pia mater from the gray matter on which it lies. Other meningeal
structures, such as the dura mater, may appear denser than brain, especially if there is
some calcification in the membranes.

(Q.192) Duplex ultrasonography combines


A-mode imaging and pulse-wave Doppler examination
(a)
B-mode imaging and pulse-wave Doppler examination
(b)
M-mode imaging and power Doppler examination
(c)
M-mode imaging and waveform analysis
(d)
Your Response
:
Correct Answer
B
:
B-mode imaging and pulse-wave doppler examination
Exp:
An objective assessment of the severity of disease is obtained by noninvasive
techniques. These include digital pulse volume recordings, doppler flow velocity
waveform analysis, duplex ultrasonography (which combines b-mode imaging and
pulse-wave doppler examination), segmental pressure measurements, transcutaneous
oximetry, stress testing (usually using a treadmill), and tests of reactive hyperemia.

(Q.193) In CT, the attenuation values are measured in Hounsfield units (HU). An attenuation value of 0 (zero)
HU corresponds to:

(a)
(b)
(c)
(d)
Your Response
:
Correct Answer
:
Exp:

Water
Air
Very dense bone structures
Fat

A
(water)
Ref. Christensens physics of diagnostic radiology 4th ed. 308

CT number/hu value:
Basic principle of CT is linear attenuation of x-rays.

For unenhanced CT, there is an essentially linear relationship between


voxel signal intensity (image brightness) and the x-ray linear attenuation
coefficient (), which is scaled relative to air and water and converted to
an integer.
This is expressed in hounsfield units (HU), which range from 1000 to

+4000.

(Q.194) The most accurate preoperative local staging of esophageal, pancreatic, and rectal
malignancies?

(a)
(b)
(c)
(d)
Your Response
:
Correct Answer
:
Exp:

Endoscopic USG
Dynamic CECT
CEMRI
SPECT scan

A
Endoscopic USG
Endoscopic ultrasound (EUS) EUS utilizes high-frequency ultrasound
transducers incorporated into the tip of a flexible endoscope.
Ultrasound images are obtained of the gut wall and adjacent organs,
vessels, and lymph nodes.
By sacrificing depth of ultrasound penetration and bringing the
ultrasound transducer close to the area of interest via endoscopy, very
high resolution images are obtained.

EUS provides the most accurate preoperative local staging of


esophageal, pancreatic, and rectal malignancies, although it
does not detect most distant metastases. EUS is also highly
sensitive for diagnosis of bile duct stones, gallbladder disease,
submucosal gastrointestinal lesions, and chronic pancreatitis. Fineneedle aspiration of masses and lymph nodes in the posterior
mediastinum, abdomen, and pelvis can be performed under EUS
guidance
(Q.195) Which of the following can best evaluate the extent of the injury to the
sphincter muscles in patients with incontinence?
(a)
Anal manometry
(b)
Endoanal ultrasound
(c)
NIRS
(d)
Pudendal nerve terminal motor latency (PNTML)
Your Response
:
Correct Answer
D
:
Exp:
Pudendal nerve terminal motor latency (PNTML)
The evaluation of fecal incontinence should include a thorough
history and physical examination, anal manometry, pudendal
nerve terminal motor latency (pntml), And endoanal ultrasound.

Anal manometry determines the resting and squeeze pressures


within the anal canal. Pudendal nerves studies evaluate the
function of the Nerves innervating the anal canal. Stretch injuries
to these nerves will result in a delayed response of the sphincter
muscle to a stimulus, indicating a prolonged Latency.
Finally, ultrasound will evaluate the extent of the injury to the
sphincter muscles. Only pntml has been able to predict outcome
following surgical Intervention.
(Q.196) The procedure of choice to confirm the presence of perirenal collections of
urine, blood, or lymph in a transplanted kidney is?
Renogram
(a)
USG
(b)
MRI
(c)
IVP
(d)
Your Response
:
Correct Answer
B
:
USG
Exp:
Arteriography and radioactive iodohippurate sodium
renograms of the transplanted kidney may be useful in
ascertaining changes in the renal vasculature and in renal
blood flow, even in the absence of urinary flow.
Thrombosis of the renal vein occurs rarely; it may be reversible if
caused by technical factors and intervention is prompt. Diagnostic
ultrasound is the procedure of choice to rule out urinary
obstruction or to confirm the presence of perirenal collections of
urine, blood, or lymph.
When renal function has been good initially, a rise in the serum
creatinine level is the most sensitive and reliable indicator of
possible rejection and may be the only sign.
(Q.197) Radiological pattern most common in nosocomial pneumonia?
Bronchopneumonia
(a)
Lobar pneumonia
(b)
Respiratory bronchiolitis
(c)
No specific pattern
(d)
Your Response
:
Correct Answer
A
:
Bronchopneumonia
Exp:
Ref: Harrison 18,page2131

In VAP, respiratory bronchiolitis may precede the development of a


radiologically apparent infiltrate.
Because of the microaspiration mechanism, a bronchopneumonia pattern is
most common in nosocomial pneumonias, whereas a lobar pattern is more
common in bacterial CAP.
Despite the radiographic appearance, viral and Pneumocystis pneumonias
represent alveolar rather than interstitial processes.

(Q.198) Which is the second most common abnormal USG FINGING that is indicative
of chromosomal anomaly ?
(a)
Cystic hygroma
(b)
Omphalocele
(c)
Congenital heart disease
(d)
Choroid plexus cyst
Your Response
:
Correct Answer
B
:
Exp:
Omphalocele
Ref: Harrison 18th table 62-1. Page 513
cystic hygroma 68%
omphalocele 39%
choroid plexus cyst 5%
congenital heart disease 30 %

Table 62-1 Frequency of Chromosome Abnormalities,


Identified on the Basis of Abnormal Ultrasound Findings

Ultrasound Finding
Abnormal ultrasound
(nonspecific)
Omphalocele
Cystic hygroma
Congenital heart disease
Choroid plexus cyst
(Q.199) Double PCL sign on MRI knee?
(a)
Lateral meniscus tear
(b)
Medial meniscus tear
(c)
PCL tear
(d)
ACL tear
Your Response
:

Chromosomal Abnormalities
(Frequency)
Average, Range in Different
%
Studies, %
16
1335
39
68
30
5

2654
4678
840
410

Correct Answer
B
:
Exp:
Medial meniscus tear
Double PCL sign on MRI knee is highly suggestive of medial
meniscal injury.
(Q.200) Which of the following is false about CT?
Imaging modality of first choice for suspected acute subarachnoid
(a)
hemorrhage.
(b)
Best for diagnosing Soft tissue pathology.
(c)
Investigation of choice for skull trauma.
(d)
Calcification is better visualized on CT than on MRI.
Your Response
:
Correct Answer
B
:
Exp:
Best for diagnosing Soft tissue pathology.
CT is the primary study of choice in the evaluation of an acute change in mental
status, focal neurologic findings, acute trauma to the brain and spine, suspected
subarachnoid hemorrhage, and conductive hearing loss. CT is not good for soft
tissue pathology.

Anda mungkin juga menyukai